You are on page 1of 554
princiles of refrigeration R.J.DOSSAT id Peete ® een | ce E ane 5 can & H § oes Bie suam |d = or iononin| gg Sa pPaae ne A \ } aan A ue @ 5 y 5 Coo Src EE EE Ta pes Eee freer tees Renter tt Carrere] comers, Coe ressure rns => Wiley International Edition PRINCIPLES OF REFRIGERATION ROY J. DOSSAT, Associate Professor of Refrigeration and Air Conditioning, University of Houston, Houston, Texas, JOHN WILEY & SONS, INC. NEW YORK and LONDON TOPPAN COMPANY, LTD. TOKYO, JAPAN Authorized reprint of the edition published by John Wiley & Sons, Inc., New York and London, Copyright © 1961 by John Wiley & Sons, Inc. Al Rights Reserved. No part of this book may be reproduced in any form without the written permission of John Wiley & Sons, Ine. Wiley International Edition This book is not to be sold outside the country to which it is consigned by the publisher. Library of Congress Catalog Card Number: 61:153%6 Printed in Singapore by Toppan Printing Co. (S) Pe. Lud. Preface This textbook has been written especially for use in programs where a full curriculum in refrigeration is offered. However, the material covered and the method of presentation are such that the text is also suitable for adult evening classes and for on-the-job training and selfinstruction. Furthermore, the material isso arranged and sectionalized that this textbook is readily adaptable to any level of study and to any desired method or sequence of presentation. Despite a rigorous treatment of the thermodynamics of the cycle, application of the calculus is not required nor is an extensive background in physics and thermodynamics presupposed. ‘The first four chapters deal with the funda- ‘mental principles of physics and thermodynamics upon which the refrigeration toased. For those who are already familiar with these fundamentals, the chapters will serve as review or reference material ‘Chapter 21 treats electric motors and control circuits as they apply to refrigera- tion and air conditioning systems, This material is presented from the viewpoint ‘of practical application, the more mathematical approach being left to companion electrical courses. Throughout this textbook emphasis is placed on the cyclic nature of the refrigeration system, and each part of the system is carefully examined in relation to the whole. Too, care is taken continually to correlate theory and practice through the use of manufacturer's catalog data and many sample problems. To this end, certain pertinent catalog data are included. Roy J. Dossat July, 1961 Acknowledgments Most of the material in this textbook is based on information gathered from publications of the American Society of Heating, Refrigerating, and Air Condition- ing Engineers and of the following equip- ment manufacturers: ‘Acme INDusTRIES, INC. ‘Atco VALVE ComPANY ANACONDA METAL HOSE Division, THE ‘AMERICAN BRASS COMPANY ‘CowtRoLs COMPANY OF AMERICA Dean Probucts, INC. Derrorr ConTRoLs Division, AMERICAN RADIATOR & STANDARD SANITARY CORPORATION Derrorr Ice MACHINE COMPANY ous REFRIGERATING COMPANY Dunsam-Bust, INC. INGERSOLL-RAND ComPANy KENNARD Division, AMERICAN AIR FILTER Company, INC. KRAMER TRENTON CoMPany McQuay, INc. ‘Tue MaRLey ComPANy Mansi INSTRUMENT COMPANY ‘Muriter Brass COMPANY PENN ConrROLS, INC. Recoup ConPoraTion ‘SPORLAN VALVE COMPANY ‘TecumseH Provucts CoMPany ‘TRANTER MANUFACTURING, INC. ‘TUBULAR EXCHANGER MANUFACTURERS ASSOCIATION, INC. ‘TYLER REFRIGERATION CorrorATion ‘THe VILTER MANUFACTURING COMPANY WorTHINGTON CORPORATION ‘Yorx CORPORATION, SUBSIDIARY OF BORO- WARNER CORPORATION EDwAnne Eneneseinag Coaronanion Appreciation is expressed to all these EL bu Powr pe Newouns & Courany organizations for their contributions in the FReezino EQUIPMENT SatEs, INC. form of photographs and other art work, Frick Company and for granting permission to reproduce Genenat ContRois Company proprietary data, without which this text- Generat Euectric COMPANY book would not have been possible. Hatsreap & MITCHELL, Contents I. Pressure, Work, Power, Energy | 2. Matter, Internal Energy, Heat, Temperature 10 3. Thermodynamic Processes 24 4, Saturated and Superheated Vapors 43 5. Psychrometric Properties of Air 57 6. Refrigeration and the Vapor Compression System 71 7. Cycle Diagrams and the Simple Saturated Cycle 89 8. Actual Refrigerating Cycles 107 9. Survey of Refrigeration Applications 121 10. Cooling Load Calculations 144 U1. Evaporators 164 12, Performance of Reciprocating Compressors 203 13, System Equilibrium and Cycling Controls 225 14, Condensers and Cooling Towers 244 15, Fluid Flow, Centrifugal Liq Pumps, Water and Brine Piping 274 16, Refrigerants 284 17, Refrigerant Flow Controls 298 wu vill PRINCIPLES OF REFRIGERATION 18. Compressor Construction and Lubrication 334 19. Refrigerant Piping and Accessories 365 20. Defrost Methods, Low-Temperature Systems, and Multiple Temperature Installations 388 21. Electric Motors and Control Circuits 407 Tables and Charts 430 Index 535 Pressure, Work, Power, Energy 14. Force. A force is defined as a push or a pull, It is anything that has a tendency to set 8 body in motion, to bring a moving body to rest, of to change the direction of motion. A force may also change the size or shape of a body. ‘That is, the body may be twisted, bent, stretched, , or otherwise distorted bby the action of a force. ‘The most familiar force is weight. The weight of a body is a measure of the force exerted on the body by the gravitational pull of the earth (Fig. 1-1) ‘There are many forces other than the force of gravity, but all forces are measured in weight units. Although the most commonly used unit of force measure is the pound, any unit of ‘weight measure may be used, and the particular unit used at any time will usually depend on the magnitude of the force to be measured. 1-2, Pressure. Pressure is the force exerted per unit of area, -It may be described as a measure of the intensity of a force at any given point on the contact surface. Whenever a force is evenly distributed over a given area, the pressure at any point on the contact surface is ‘the same and can be calculated by dividing the total force exerted by the total area over which the force is applied. This relationship is expressed by the following equation F Pao an where P = the pressure expressed in units of F per unit of A F = the total force in any units of force ‘A = the total area in any units of area 1.3. Measurement of Pressure. As indi- cated by equation 1-1, pressure is. measured in units of force per unit of area. Pressures are ‘most frequently given in pounds per square inch, abbreviated psi. However, pressure, like force, a8 a matter of convenience and depending con the magnitude of the pressure, may be stated. in terms of other units of force and area, such ‘8s pounds per square foot, tons per square foot, ‘grams per square centimeter, etc. Example I-1, A rectangular tank, measur- ing 2 f by 3 ft at the base, is filled with water. If the total weight of the water is 432 Ib, deter- ‘mine the exerted by the water on the ‘bottom of the tank in (@) pounds per square foot (©) pounds per square inch Solution @ Area of tank base 2x3 =6sqit Total weight of water = 4321 Applying Equation 1-1, P 2 = Nps (6) Area of the tank base 24 x 36 = 864 5q in. Total weight of water = 4321 Applying Equation 1-1, P a = 05 psi The problem described in Example 1-1 is itlustrated in Fig. 1-2. Notice that the pressure con the bottom of the tank in pounds per square foot is to the downward force exerted by the weight of a column of water having a cross section of one square foot, where- as the pressure in pounds per square’ inch is equivalent to the downward force exerted by @ column of water having a cross section of sq in. Further, since there are 14 9q in. in 1 sq ft, the force exerted per square foot is 144 times as great as the force exerted per square inch. 1-4, Atmospheric Pressure. The carth surrounded by an envelope of atmosphere or air which extends upward from the surface of the earth to a distance of some 50 mi or more. Air has weight and, because of its weight, exerts a 2 PRINCIPLES OF REFRIGERATION 4 [Spring scale Pointer —] ight Fig. tol, Because of gravity, ‘exerts a\downward force of 7 It suspended welght pressure onthe surface of theearth. Thepressure exerted by the atmosphere is known as stmos- pressure. ‘The weight of a column of air having a cross section of 1 sq in. and extending from the surface (of the earth at sea level to the upper limits of the ‘atmosphere is 14.696 Ib. Therefore, the pressure con the surface of the earth at sea level resulting from the weight of the atmosphere is 14.696 psi (14.7), This is understood to be the normal or standard atmospheric pressure at sea level and is sometimes referred to as a pressure of one atmosphere, Actually, the pressure of the ‘atmosphere does not remain constant, but will usually vary somewhat from hour to hour depending upon the temperature, water vapor content, and several other factors. ‘Because of the difference in the height of the column, the weight of a column of air of given ‘cross section will be less when taken at an altitude of one mile above sca level than when. taken at sea level. Therefore, it follows that atmospheric pressure decreases as the altitude increases. 15. Barometers. Barometersareinstruments used to measure the pressure of the atmosphere and are of several types. A simple barometer which measures atmospheric pressure in terms of the height of a column of mercury can be constructed by filling with mercury a hollow glass tube 36 in. or more long and closed at one end. The mercury is held in the tube by placing the index finger over the open end of the tube while the tube is inverted in an open dish of ‘mercury. When the finger is removed from the tube, the level of the mercury in the tube will fall, leaving an almost perfect vacuum at the closed end. ‘The pressure exerted downward by the atmosphere on the open dish of mercury will cause the mercury to stand up in the evacuated tube to a height depending upon the amount of, exerted. ‘The height of the mercury column in the tube is a measure of the pressure exerted by the atmosphere and is read in inches ‘of mercury column (abbreviated in. Hg). The normal pressure of the atmosphere at sea level (4.696 psi) pressing down on thedish of mercury will cause the mercury in the tube to rise to Fig. 12. OF the total weight of the water container, that part which I exerted on | sqft area [nthe pressure in pounds per square foot. Likewise, that part which is exerted on a | aq In. area isthe pressure In pounds per square inch. height of 29.921 in, (Fig. 1-3). A column of ‘mercury 29,921 in. high is, then, a measure of a equivalent to 14.696 psi. By dividing 29.921 in. Fig by 14.696 pai, itis determined that a pressure of 1 psi is equivalent to a pressure of 2.036 in. Hg. Therefore, 1 in. Hg equals 1/2.036, or 0.491 psi, and the following equa- tions are established: in. Hg = ES; 2) and psi = in. Hg x 0.491 a3) Example 1-2, What is the pressure of the atmosphere in psi if a barometer reads 30.2 in. Hg? Soliton. Applying Equation 1-3, P= 502% 0491 = 1483 psi Example 1-3. In Fig. 1-3, how high will the mercury stand in the tube when the atmos- pheric pressure is 14.5 psi? Solution. Applying Equation 1-2, Pas oasi = 29.53 in. Hg 1-6, Pressure Gages. Pressure gages are instruments used to measure the fluid pressure (ither gaseous or liquid) inside a closed vessel. Pressure gages commonly used in the refriger- ation industry are of two types: (1) manometer and (2) bourdon tube. Manometers. The manometer type gage utilizes a column of liquid to measure the pressure, the height of the column indicating the magnitude of the presture. The liquid used inmanometersis usually either water or mercury. ‘When mercury is used, the instrument is known as a mercury manometer or mercury gage and, when water is used, the instrument is a water manometer or water gage. ‘The simple barom- eter described previously is a manometer type instrument. ‘A simple mercury manometer, illustrated in Figs. 1-4a, 1-46 and 1-4c, consists of a U-shaped. ass tube open at both ends and partially filled with mercury. When both legs of the U-tube fare open to the atmosphere, atmospheric pres- sure is exerted on the mercury in both sides of the tube and the height of the two mercury columns is the same. The height of the two mercury columns at this postion is marked the zero point of the scale and the scale is cali PRESSURE, WORK, POWER, ENERGY 3 brated in inches to read the deviation of the ‘mercury columns from the zero condition in either direction (Fig. 14a). When in use, one side of the U-tube is connected to the vessel whose pressure is to be measured. The pressure in the vessel, acting on ‘one leg of the tube, is opposed by the atmos- pheric pressure exerted on the open leg of the tube. If the pressure in the vessel is greater than that of the atmosphere, the level of the mercury on the vessel side of the U-tube is depressed while the level of the mercury on the ‘open side of the tube is raised an equal amount (Fig. 1-46), If the pressure in the vessel is less than that of the atmosphere, the level of the ‘mercury in the open leg of the tube is depressed while the level of the mercury in the leg con- nected to the vessel is raised by an equal amount Fig. 1-4). In either case, the difference in the Iheights of the two mercury columns is a measure of the difference in pressure between the total pressure of the fluid in the vessel and the pressure of the atmosphere. In Fig. 1-45, the level of the mercury is 2 in. Dich of mercury Fig. 1-3. The pressure exerted by the weight of the ‘atmosphere on the open dish of mercury causes the mercury to stand up Into the tube. The magnitude of the pressure determines the height of the mercury column, 4 PRINCIPLES OF REFRIGERATION Fig. I-4a, Simple U-tube manometer. Since both legs of the manometer are open to che atmosphere and areat the same pressure, the level ofthe mercury Is the same In both sides. Vessel pressure ‘Hin Hg Fig. 1-4b, Simple manometer indicates. that the vessel pressure exceeds the atmospheric pressure by 4in, He. below the zero point in the side of the U-tube connected to the vessel and 2 in. above the zero point in the open side of the tube. This indi- ‘cates that the pressure in the vessel exceeds the pressure of the atmosphere by 4 in. Hg (1.96 psi). In Fig. 1-4c, the level of the mercury is depressed the side of the tube open to the atmosphere and raised 2 in. in the side con- nected to the vessel, indicating that the pressure in the vessel is 4 in. Hg (1.96 psi) below (less than) atmospheric, Pressures belowatmospheric are usually called “vacuum” pressures and may bbe read as “inches of mercury, vacuum.” Manometers using water as the measuring fluid are particularly useful for measuring very small pressures. Because of the difference in the density of mercury and water, pressures so slight that they will not visibly affect the height of a mercury column will produce easily detectable variations in the height of a water column. Atmospheric pressure, which will support a column of mercury only 29.921 in. high, will lift a column of water to a distance of approximately 34 ft. A pressure of 1 psi will raise a column of water 2:31 ft or 27.7 in, and a Fig. Ie. Manometer indicates that the vessel pressure Is 4 In. Hg less than the stmospherle pressure of 30 In. Hg. Fig. 1-5. Bourdon tube gage mechanism. (Courtesy Marsh instrument Company.) pressure of only 0.036 pai is sufficient to support column of water 1 in. high. Hence, 1 in. of water column is equivalent to 0.036 psi. Table 1-1 gives the relationship between the various units of pressure measurement. 148, Bourdon Tube Gages. Because of the excessive length of tube required, gages of the ‘manometer type are not practical for measuring Pressures above 15 psi and are more or less PRESSURE, WORK, POWER, ENERGY 5 inches of mercury (Fig. 1-66). In many cases, single gages, known as “compound” gages, are designed to measure pressures both above and bbelow atmospheric (Fig. 1-6c). Such gages are calibrated to read in psi above atmospheric and in inches of mercury below atmospheric. 1.9. Absolute and Gage Pressures. Absolute pressure is understood to be the “total” or “true” pressure of a fluid, whereas gage pressure © ® o Fig. 1-6. Typical Bourdon tube gages. (0) Pressure gage. (b) Vacuum gage. (¢) Compound gage. (Courtesy Marsh Instrument Company.) limited to the measurement of relatively small pressures in air ducts, etc. ‘Gages of the bourdon tube type are widely used to measure the higher pressures en- ‘countered in refrigeration work. ‘The actuating mechanism of the bourdon tube gage is illus: trated in Fig. 1-5. The bourdon tube, itself, is a curved, elliptical-shaped, metallic tube which tends to straighten as the fluid pressure in the tube increases and to curl tighter as the pressure decreases, Any change in the curvature of the tube is transmitted through a system of gears to the pointer. The direction and magnitude of the pointer movement depend on the direc- tion and magnitude of the change in the curva- ture of the tube. ‘Bourdon tube gages are very rugged and will measure pressures either above or below atmospheric pressure. Those designed to ‘measure pressures above atmospheric are known as “pressure” gages (Fig, 1-6a) and are gener- ally calibrated in psi, whereas those designed to read pressures below atmospheric are called “vacuum” gages and are usually calibrated in is the pressure as indicated by gage. It is important to understand that gages are cali- brated to read zero at atmospheric pressure and that neither the manometer nor the bourdon tube gage measures the “total” or “true” Pressure of the fluid in a vessel; both measure ‘only the difference in pressure between the total pressure of the fluid in the vessel and the atmos- heric pressure. When the fluid pressure is greater than the atmospheric pressure, the absolute pressure of the fluid in the vessel is determined by adding the atmospheric pressure to the gage pressure, and, when the fluid pressure is less than atmospheric, the absolute Pressure of the fluid is found by subtracting the {gage pressure from the atmospheric pressure. The relationship between absolute pressure and ‘age pressure is shown graphically is Fig. 1-7. Example 1-4, A pressure gage on a refrigerant condenser reads 120 psi. What is the absolute pressure of the refrigerant in the condenser’? Solution. Since the barometer reading is not given, it is assumed that the atmospheric 6 PRINCIPLES OF REFRIGERATION age ‘Absolite pressure pressure «6 597 “0 547 % 497 2 —— a7 25-1 Fressures stove | 297 ao-|_simowonercinst_| 4 18. 237 w+ 247 5 197 o—{ Atmospheric pressure | 2992/0. Hg 5 25 (47 pa 10-]—Pressres below —t 20 15-]— atmospheric in —f 15 2 in He’ 10 047 pa 5 29598 ng o Fig. 1-7, Relationship between absolute and gage Pressures. pressure is normal at sea level, 14.696 psi, and, since the of the refrigerant is above atmospheric, the absolute pressure of the refrigerant is equal to the gage pressure plus the atmospheric pressure. Gage pressure in psi =120 Atmospheric pressure in psi = 14.696 Absolute pressure of oe refrigerant = 134,696 psi Example 1-5. A compound gage on the suction side of a vapor compressor reads 3 in, Hg, whereas a barometer nearby reads 29.6 in, Hg. Determine the absolute pressure Of the vapor entering the compressor. Solution. Since the pressure of the vapor entering the compressor is less than atmospheric, the absolute pressure of the vapor is computed bby subtracting the gage pressure from atmos- pheric pressure. | Atmospheric pressure in “Gage presen in. Hg = 50 __ Absolute pressure in in, ‘Absolute pressure Example 1-6. During compression the pressure of a vapor is increased from 10 in. Hg gage to 125 psi gage. Calculate the total increase. in pressure in psi. Solution. Since the pressure increases from 10 in, Hg below atmospheric to 125 psi above atmospheric, the total increase in pressure is the sum of the two pressures. Initial pressure = 10 in. Hg Initial pressure inpsibelow 10 x 0.491 atm = 491 psi "Final pressure in psi above atmospheric = 125 psi Total increase in pressure = 129.91 psi Absolute pressure in psi is abbreviated psia, ‘whereas gage pressure in psi is abbreviated psig, 10. Work. Work is done when a force acting on a body moves the body through a distance. The amount of work done is the product of the force and the distance through which the force acts. This relationship is shown by the following equation: WeFxt a4) where F = the force applied in any units of force 1 = the distance through which the force acts in any linear unit W = the work done expressed in units of force and linear measure ‘The work done is always expressed in the same unit terms used to express the magnitude Of the force and the distance. For instance, if the force is expressed in pounds and the dis- tance in feet, the work done is expressed in foot-pounds. The foot-pound is the most frequently used unit of work measure, Example 1-7. A ventilating fan 515 holed to he root of bung S00 ‘above the level of the ground. How much work is done? ‘Solution. By applying Equation 1-4, the weight of the fan = 315 Ib Distance through which the fan is hoisted = 200 ft ‘Work done 315 x 200 = 63,000 ft-lb Ll. Power. Power is the rate of doing ‘work, That is itis the work done divided by the time required to do the work. The unit of power isthe horsepower. One horsepower is defined as the power required to do work at the rate of 33,000 feb per minute or (33,000/60) 550 fe1b per second. The power required in horsepower may be found by either of the following equa- tions: Ww HP = 35000 xt where Hp = the horse W = the work done in foot-pounds 1 = the time in minutes as) Hp = 350 xt a where ¢ = the time in seconds Example 1-8, In Example 1-7, if the time wired to hoist the fan to the roof of the building is $ minutes, how much horsepower is required? Solution. Total work done = 63,000 ft-lb Time required to do the work = 5 min i 63,000 eee ean 35,000 x3 = 0.382 hp 1412, Energy. In order to do work or to cause motion of any kind, energy is required. A body is said to possess energy when it has the capacity for doing work. Hence, energy is described as the ability to do work. ‘The amount of energy required to do a given amount of work is always equal to the amount of work done and the amount of energy a body possesses is equal to the amount of work a body can do in passing from one condition or position to another. Energy may be possessed by a body in either or both of two basic kinds: (1) kinetic and (2) potential. 3. Kinetic Energy. Kinetic energy is the energy a body possesses as a result ofits motion or velocity. For instance, a hammer swinging through an arc, a bullet speeding toward a target, and the moving parts of machinery all hhave kinetic energy by virtue of their motion. ‘The amount of kinetic energy a body possesses isa function of its mass and its velocity and may be determined by the following equation: Mxv Ka an PRESSURE, WORK, POWER, ENERGY 7 ‘where = the kinetic energy in foot-pounds ‘M = the weight of the body in pounds V = the velocity in feet per second (fps) § ~ gravitational constant 32.174 fr/sec) Example 1-9 An automobile weighing 3500 Ib is moving at the rate of 30 mph. What is its kinetic energy? ‘Solution. Velocity 5280 ffmi_x 30 mi in fps V ~~ 3600 secfhr = 44 fps Applying Equa- 3500 Ib x (44 fps)* tion 1-7, the kinetic “2 32.174 fxfeeo™ energy K = 105,302 feb Potential Energy. Potential energy is the energy a body ‘because of its position or configuration. The amount of work ‘8 body can do in passing from a given position or condition to some reference position or condition is a measure of the body's potential energy. For example, the driving head of a pile river has potential energy of position when raised to some distance above the top of a piling, If released, the driving head can do the ‘work of driving the piling. A compressed steel spring or a stretched rubber band possesses potential energy of configuration, Both the steel spring and the rubber band have the ability to do work because of their tendency to return to their normal condition. ‘The potential energy of a body may be evalu- ated by the following equation: PaMxz a8) ‘where P = the potential energy in foot-pounds ‘M = the weight of the body in pounds Z =the vertical distance above some datum or reference Example I-10, Ten thousand gallons of water are stored in a tank Jocated 250 ft above the ground. Determine the potential energy of the water in relation to the ground. Solution. The weight of the ' as) water in 10,000 gal x 8. Freres egg etone Applying Equation 1-8, the potential 83,300 Ib x 250 ft energy P = 20,825,000 ft-lb 8 PRINCIPLES OF REFRIGERATION 1-15. Energy as Stored Work. Before a body can possess energy, work must be done on. the body. ‘The work which is done on a body to give the body its motion, position, or con- figuration is stored in the body as energy. Hence, energy is stored work. For instance, ‘work must be done to stretch therubberband,to compress the steel spring, or to raise the driving head of a pile-driver to a position above the piling. In any case, the potential energy stored is equal to the work done. ‘The amount of energy a body possesses can be ascertained by the amount of work done on the body to give the body its motion, position, or configuration. For example, assume that the driving head of a pile-driver weighing 200 Ibis raised to a position 6 ft above the top of a piling. The work done in raising the driving head is 1300 fb 200 Ib x 6 ft) Therefore 1200 ft-lb of energy are stored in the driving head in its raised position and, when released, neglecting fiction, the driving-head will do 1200 ft-1b of work on the piling. 1-16, Total External Energy. The total external energy of a body is the sum of its kinetic and potential energies. Example lll. Determine thefts external energy of an airplane weighing 10,000 Ib and Sing no 1 above the ground ata sped of Solution. Apply- tng Equation 7, the kinetic energy 10,000 Ib x (440 fps), xK “7 x 32174 fisec™ a = 30,086,436 fib Equation 1-8, the 10,000 Ib x 6000 ft potential energy P = 60,000,000 f-Ib ‘Adding, the total external ‘energy = 90,086,436 ft-lb J. Law of Conservation of Energy. The First Law of Thermodynamics states in effect that the amount of energy is constant. None can be either created or destroyed. Energy is expended only in the sense that itis converted from one form to another. 1-18, Forms of Energy. All energy can be classified as being of either of the two basic kinds, kinetic or potential. However, energy ‘may appear in any one of a number of different forms, such as mechanical energy, electrical ‘energy, chemical energy, heat energy, etc., and is readily converted from one form to another. Electrical energy, for instance, is converted into hheat energy in an electric toaster, heater, or range. Electrical energy is converted into mechanical energy in electric motors, solenoids, and other electrically operated mechanical devices. Mechanical energy, chemical energy, and heat energy are converted into electrical energy in the generator, battery, and thermo- couple, respectively. Chemical energy is con- verted into heat energy in chemical reactions such as combustion and oxidation. These are ‘only a few of the countless ways in which the transformation of energy can and does occur. There are many fundamental relationships which exist between the various forms of energy and their transformation, some of which are of particular importance in the study of refrigera~ tion and are discussed in detail later. PROBLEMS 1. The cooling tower on the roof of a building weighs 1360 1b when filled with water. If the basin of the tower measures 4 ft by 5 ft, what is the pressure exerted on the roof (@) in pounds per square foot? Ans. 68 psf. @ in pounds per square inch? ‘Ans, 0.472 psi. 2. If the atmospheric pressure is normal at sea level and a gage on an R-12 condenser reads es gece eee em Freon in the condenser in pounds per inch? vans. 148.7 pa, ‘What is the total force exerted on the top of ston if the area of the cylinder bore sq in. and the pressure ofthe gas in the cylinder is 150 psi? Ans. 750 1b. 4, A barometer reads 10 in. Hg. What is the atmospheric pressure in psi? Ans. 4.91 psi. ‘5. A barometer on the wall reads 29.6 in. Hg while a gage on the tank of an air indicates 165 psi. What is the absolute of the air in the tank in pounds foot? Ans. 119.53 ptia. 6 A gage on the suction inlet of a compressor reads 10 in, Hg. Determine the absolute pres- sure of the suction vapor in psi. Ans. 9.79 psia. 7. A gage on the suction side of a refrigerati reads 5 in. Hy. If a gage on the side of the compressor reads 122 psi, what is the increase in pressure the ‘compression? ‘Ans, 124.46 psi. & An electric motor weighing 236 Ib is hoisted to the roof of a building in 2 min. If the roof is 125 ft above the (@) How much work is done? ‘Ans, 29,500 ft-lb @) Neglecting friction and other losses, what is the horsepower required? gue ans. 0447 hp 9. Compute the kinetic energy of an automobile ‘weighing 3000 Ib and moving at a speed of 75 mph. Ans, 567,188 ft-lb 10, What is the total external of the automobile in Problem 9 if the automobile is sled ? aa Meno 18 000,000 feb ‘11, What is the total ‘of 8000 gal of water ‘tank and located PRESSURE, WORK, POWER, ENERGY 9 a mean distance of 135 ft above the ground? ‘Ans. 8,996,400 ft-lb 12, Water in a river 800 ft above sea level is flowing at the rate of S mph. Calculate the sum of the kinetic and potential energies per pound of water in reference to sea level. ‘Ans. 800.84 fel 13. A water pump delivers 60 gal per minute of water to a water tank located 100 ft above the level of the pump. If water weighs 8.33 Ib per gallon and if the friction of the pipe and Other losses are , (@ How much work is done? Ans, 49,980 ft-lb (©) Compute the horsepower required. ‘Ans. 1.5 hp. 2 Matter, Internal Energy, Heat, Temperature 21. Heat. Heat is a form of energy. This is evident from the fact that heat can be con- ‘verted into other forms of energy and that other forms of energy can be converted into heat. However, there is some confusion as to exactly what energy shall be termed heat energy. Popular usage has made the concept of heat as internal or molecular energy almost universally accepted. Because of this, referring to heat as internal energy is almost unavoidable at times. On the other hand, from a strictly thermo- dynamic point of view, heat is defined as energy in transition from one body to another as. & result of a difference in temperature between the two bodies. Under this concept, all other energy transfers occur as work. Both these concepts of heat will evolve in this and the following chapters. The term heat will be used hereafter in this book in either sense. 22. Matter and Molecules. Everything in ‘the universe that has weight or occupies space, all matter, is composed of molecules. Mole- cules, in turn, are made up of smaller particles called atoms and atoms are composed of stil smaller particles known as electrons, protons, neutrons, etc. The study of atoms and sub- ‘atomic particles is beyond the scope of this book and the discussion will be limited for the most part to the study of molecules and their behavior. 10 The molecule is the smallest, stable particle ‘of matter into which a particular substance can bbe subdivided and still retain the identity of the original substance. For example, a grain of table salt (NaCI) may be broken down into individual molecules and each molecule will bbe a molecule of salt, the original substance. However, all molecules are made up of atoms, 80 that it is possible to further subdivide a molecule of salt into its component atoms. But, a molecule of salt is made up of one atom of sodium and one atom of chlorine. Hence, if a molecule of salt is divided into its atoms, the atoms will not be atoms of salt, the original substance, but atoms of two entirely different substances, one of sodium and one of chlorine, ‘There are some substances whose molecules ‘are made up of only one kind of atoms. The molecule of oxygen (0,), for instance, is ‘of two atoms of oxygen. If a mole- ccule of oxygen is divided into its two component atoms, each atom will be an atom of oxygen, the original substance, but the atoms of oxygen will not be stable in this condition. They will not remain as free and separate atoms of oxygen, but, if permitted, will either join with atoms or ‘molecules of ancther substance to form a new ‘compound or rejoin each other to form again a molecule of oxygen. tis assumed that the molecules that make up fa substance are held together by forces of mutual attraction known as cohesion. These forces of attraction that the molecules have for each other may be likened to the attraction that exists between unlike electrical charges or ‘between unlike magnetic poles. However, despite the mutual attraction that exists between the molecules and the resulting influence that ‘each molecule has upon the others, the mole- ceules are not tightly packed together. There is a certain amount of space between them and they are relatively free to move about. The mole- cules are further assumed (0 be in a state of rapid and constant vibration or motion, the rate and extent of the vibration or movement being determined by the amount of energy ‘they possess. 2-3. Internal Energy. It has been pre- viously stated that energy is required to do work or to cause motion of any kind. Molecules, like everything else, can move about only if they possess energy. Hence, a body has internal MATTER, INTERNAL ENERGY, HEAT, TEMPERATURE 11 energy as well as external energy. Whereas 1 body has extemal mechanical energy because of its velocity, position, or configuration in relation to some reference condition, it also has internal energy as a result of the velocity, position, and configuration of the molecules of the materials which make up the body. The molecules of any material may possess energy in both kinds, kinetic and potential. ‘The total internal energy of a material is the sum ofits internal kinetic and potential energies. This relationship is shown by the equation U=K+P where U = the total internal energy = the internal kinetic energy P = the internal potential energy 2-4, Internal Kinetic Energy. Internal kinetic energy is the energy of molecular motion or velocity. When heat energy flowing into a material increases the internal kinetic energy, the velocity or motion of the molecules is increased. ‘The increase in molecular velocity is always accompanied by an increase in the temperature of the material. Hence, a material's temperature is, in a sense, a measure of the average velocity of the molecules which make up the material. The more kinetic energy the ‘molecules have, the greater is their movement and the faster they move. ‘The more rapid the motion of the molecules, the hotter is the material and the more internal kinetic energy the material has. It follows, then, that if the internal kinetic energy of the material is di- inished by the removal of heat, the motion of the molecules will be slowed down or retarded and the temperature of the material will be decreased. ‘According to the kinetic theory ifthe removal ‘of heat continues until the internal Kinetic ‘energy of the material is reduced to zero, the temperature of the material will drop to Abso- lute Zero (approximately —460°F) and the motion of the molecules will cease entirely.* * It is now known that the energy is not zero at Absolute Zero. It isthe disorganization (entropy) Which diminishes to zero. Heat is sometimes defined as “disorganized energy.” Both the energy and the disorganization decrease as the temperature ‘decreases. However, the disorganization decreases faster than the energy and therefore diminishes to zer0 before the energy reaches zer0. (4) 25, States of Matter. Matter can exist in three different phases or states of aggregation: solid, liquid, or a vapor or gas. For example, water is a liquid, but this same substance can exist as ice, which is a solid, or as steam, which isa vapor or ps 2-6, The Effect of Heat on the State of Aggregation. Many materials, under the proper conditions of pressure and temperature, ‘can exist in any and all of the three physical states of matter. It will be shown presently that the amount of energy the molecules of the ‘material have determines not only the tem- perature of the material but also which of the three physical states the material will assume. any particular time. In other words, the addi- tion or removalof heat can bring about a change in the physical state of the material as well as a ‘change in its temperature. That heat can bring about a change in the ‘physical state of a material is evident from the fact that many materials, such as metals will become molten when sufficient heat is applied. Furthermore, the: phenomenon of melting ice and boiling water is familiar to everyone, Each ‘of these changes in the physical state is brought about by the addition of heat. 21. Internal Potential Energy. Internal potential energy is the energy of molecular separation or configuration. Itis the energy the molecules have as a result of their position in relation to one another. The greater the degree of molecular separation, the greater is the inter- nal potential energy. ‘When a material expands or changes its physical state with the addition of energy, a it of the molecules takes place which increases the distance between them. Inasmuch as the molecules are attracted to one. another by forces which tend to pull them to- gether, internal work must be done in order to Separate further the molecules against their attractive forces. An amount of energy equal to the amount of internal work done must flow into the material. This energy is set up in the ‘material as an increase in the internal potential energy. It is “stored” energy which is accounted for by the increase in the mean dis- tance between the molecules. The source of this energy is the heat energy supplied. It is important to understand that in this instance the energy flowing into the material 12 PRINCIPLES OF REFRIGERATION hhas no effect on molecular velocity (internal kinetic energy); only the degree of molecular separation (the internal potential energy) is affected. 28. The Solid State. A material in the solid state has a relatively small amount of internal potential energy. The molecules of the material ae rather closely bound together by each other's attractive forces and by the force of gravity. ‘Hence, a material in the solid state has a rather rigid molecular structure in which the position of each molecule is more or less fixed and the ‘motion of the molecules is limited to a vibratory type of movement which, depending upon the amount of internal kinetic energy the molecules possess, may be either slow or rapid. Because of its rigid molecular structure, solid tends to retain both its size and its shape. A solid is not compressible and will offer considerable resistance to any effort to change its shape. 29. The Liquid State. The molecules of a ‘material in the liquid state have more energy than those of a material in the solid state a they are not so closely bound together. Their greater energy allows them to overcome each other's attractive forces to some extent and to have mote freedom to move about. They are free to move over and about one another in such a way that the material is said to “flow.” Although a liquid is noncompressible and will retain its size, because of its fluid molecular structure, it will not retain its shape, but will assume the shape of any containing ‘vessel, IO, The Vapor or Gaseous State, The ‘molecules of a material in the gaseous state have fan even greater amount of energy than those of a material in the liquid state. ‘They have sufficient energy to overcome all restraining forces. They are no longer bound by each other's attractive forces, neither are they bound by the force of gravity. Consequently, they fly about at high velocities, continually collid- ing with each other and with the walls of the container. For this reason, a gas will retain neither its size nor its shape. It is readily com- pressible and will completely fill any container regardless of size. Further, if the gas is not stored in a sealed container, it will escape from the container and be diffused into the surround- ing air. 211, Temperature. Temperature is a prop- erty of matter. It is a measure of the level of heat intensity or the thermal pressure of a body. A high temperature indicates a high level of heat intensity or thermal pressure, and the body is said to be hot. Likewise, 2 low temperature indicates a low level of heat intensity or thermal pressure arid the body is said to be cold, 2-12, Thermometers. The most frequently ‘used instrument for measuring temperature is the thermometer. The operation of most ther- ‘mometers depends upon the property of a liquid to expand or contract as its temperature is increased or decreased, respectively. Because of their low freezing temperatures and relatively constant coefficients of expansion, alcohol and ‘mercury are the liquids most frequently used in thermometers. The mercury thermometer is the ‘more accurate of the two because its coefficient of expansion is more constant through a greater ‘temperature range than is that of alcohol. ‘However, mercury thermometers have the dis- advantage of being more expensive and more difficult to read. Alcohol is cheaper and can be colored for easy visibility. ‘Two temperature scales are in common use today. The Fahrenheit scale is used in English speaking countries, whereas the Centigrade scale is widely used in European countries as well as for scientific purposes. 2-13. Centigrade Seale. The point at which water freezes under atmospheric pressure is, taken as the arbitrary zero point on the Centi- {grade scale, and the point at which water boils. is designated as 100. The distance on the scale between these two points is divided into one ‘hundred equal units called degrees, so that the distance between the freezing and boiling points of water on the Centigrade scale is 100°. Water freezes at 0° Centigrade and boils at 100° Centi- rade. E1A Fabrenhele Scale. Atthough there is some disagreement as to the actual method used by Fahrenheit in designing the first temperature scale, it was arrived at by means similar to those described in the previous section. On the Fahrenheit scale, the point at which water freezes is marked as 32, and the point at which water boils 212. Thus, there are 180 units between the freezing and boiling points of water. The zero or reference point on the Fahrenheit scale is placed 32 units or degrees MATTER, INTERNAL ENERGY, HEAT, TEMPERATURE 13 below the freezing point of water and is assumed to represent the lowest temperature Fahrenheit could achieve with a mixture of ammonium chloride and snow. 215. Temperature Conversion. Temper ature readings on one scale can be converted to reading on the other scale by using the appro- priate of the following equations: oF = 9/5°C +32 an + C = 5/90 F - 32) @) Tt should be noted that the difference between the freezing and boiling points of water on the Fahrenheit scale is 180°, whereas the difference between these two points on the Centigrade scale is only 100°. Therefore, 100 Centigrade degrees are equivalent to 180 Fahrenheit . This establishes a relationship such that 1° C equals 9/5° F (1.8° F) and 1° F equals 5/9° C (0.555° C). This is shown graphically in Fig. 2-1. Since 0° on the Fahrenheit scale is 32° F below the freezing point of water, it is necessary to add 32° F to the Fahrenheit equi lent after converting from Centigrade. Like- wise, it is necessary to subtract 32°F from a Fahrenheit reading before converting to Centi- grade, Example 2-1. Convert a temperature read- ing of 50°C to the equivalent Fahrenheit temperature, Solution. Applying Equation 2.2, *F 9/5(50° C) + 32 = 12°F Example 22. A thermometer on the wall of a room reads 86°F. What is the room temperature in degrees Centigrade? ‘Solution. Applying Formula 2.3, the room temperature 5/9(86-32) in*c =30°C Example 23. A thermometer indicates that the temperature of a certain quantity of water is increased 45°F by the addition of hheat. Compute the temperature rise in Centi- grade degrees. Solution. Temperature rise in * F pes = 45°F Temperature rise in °C 5/945° F) =25°C 2-16. Absolute Temperature. Tempera- ture readings taken from either the Fahrenheit Boling point of water ai 100" Frezing point of water |, =" Seales coincide age L__tosotute 2891 _ay 3p Fig. 2-1. Comparison of Fahrenhelt and Centigrade temperature scales. or Centigrade scales are in respect to arbitrarily selected zero points which, as has been shown, are not even the same for the two scales. When. it is desired to know only the change in tem- perature that occurs during a process or the temperature of a substance in relation to some Known reference point, such readings are entirely adequate. However, when temperature readings are to be applied in equations dealing with certain fundamental laws, it is necessary to use temperature readings whose reference point is the true or absolute zero of tempera ture. Experiment has indicated that such a point, known as Absolute Zero, exists at approxi- mately ~460° F or —273° C (Fig. 2-1). ‘Temperature readings in reference to Abso- lute Zero are designated as absolute tempera- tures and may be in either Fahrenheit or Centigrade degrees. A temperature reading on the Fahrenheit scale can be converted to absolute temperature by adding 460° to the Fahrenheit reading. The resulting temperature is in degrees Rankine (° R). Likewise, Centigrade temperatures can be converted to absolute temperatures by adding 273 to the Centigrade reading. The resulting temperature is stated in degrees Kelvin (°K). 14 PRINCIPLES OF REFRIGERATION In converting to and from absolute tempera- tures, the following equations will apply: T= 14460 4) t=T— 460 es Tet+73 eo t=r-273 en where T= absolute temperature in degrees Rankine or Kelvin f= temperature in degrees Fahrenheit ‘or Centigrade ‘Equations 2-4 and 2-5 apply to the Rankine and Falirenheit scales, whereas Equations 2-6 and 2-7 apply to the Kelvin and Centigrade scales. ‘Hereafter in this book Rankine and Fahrenheit ‘temperatures are used unless otherwise specified. Example 24, A thermometer on the tank ofan air r indicates that the tempera- ture of the air in the tank is 95°F. Determine the absolute temperature in degrees Rankine, ‘Solution. Applying Equation 2-4, T= 98°F + 460° =555°R Example 25. The temperature of the ‘vapor entering the suction of a refrigeration ris ~20" F. Compute the tempera- ture of the vapor in degrees Rankine. Solution. Applying Equation 2-4, T = ~20°F + 460° = 440° R Example 2-6, If the temperature of a gas is 100°C, what is its temperature in degrees Kelvin? Solution. Applyi Equation 2-6, eyes 100" C + 273° = 373K. Example 27. The temperature of steam boiler is 610" R._ What is the tempera- ture of the steam on the Fahrenheit scale’ Solution. Ay Equtce ey ger — ar = 150F 2-17, Direction and Rate of Heat Flow. Heat will low from one body to another when, and only when, a difference in temperature exists between the two bodies. If the tempera- ture of the two bodies is the same, there is no transfer of heat, Heat always flows down the temperature seale from a high temperature to a low tem- perature, from a hot body to a cold body, and never in the opposite direction. Since heat is energy and cannot be destroyed, if heat is to leave one body of material, it must flow into ‘and be absorbed by another body of material whose temperature is below that of the body ‘being cooled. ‘The rate of heat transfer between two bodies, is always directly proportional to the difference in temperature between the two bodies. 2-18. Methods of Heat Transfer. The trans- fer of heat from one place to another occurs in three ways: (1) conduction, (2) convection, and (3) radiation. 219. Conduction. Heat transfer by con- duction occurs when energy is transmitted by direct contact between the molecules of a single body or between the molecules of two or more bodies in good thermal contact with each other. In either case, the heated molecules communi- ‘cate their energy to the other molecules im- mediately adjacent to them. The transfer of ‘energy from molecule to molecule by conduction, is similar to that which takes place between the balls on a billiard table, wherein all or some part of the energy of motion of one ball is trans- mitted at the moment of impact to the other balls that are struck. ‘When one end of a metal rod is heated over a flame, some of the heat energy from the heated ‘end of the rod will flow by conduction from molecule to molecule through the rod to the ‘cooler end, As the molecules at the heated end of the rod absorb energy from the flame, their ‘energy increases and they move faster and through a greater distance. The increased ‘energy of the heated molecules causes them to strike against the molecules immediately ad- Jacent to them. At the time of impact and ‘because of it, the faster moving molecules trans- mit some of their energy to their slower moving neighbors so that they too begin to move more rapidly. In this manner, energy passes from molecule to molecule from the heated end of the rod to the cooler end. However, in no case ‘would it be possible for the molecules furthest from the heat source to have more energy than ‘those at the heated end. MATTER, INTERNAL ENERGY, HEAT, TEMPERATURE 15 [As heat passes through the metal rod, the air immediately surrounding the rod is also heated bby conduction, The rapidly vibrating particles of the heated rod strike against the molecales of the air which are in contact with the rod. ‘The energy so imparted to the air molecules causes them to move about at a higher rate and communicate their energy to other nearby air ‘molecules. Thus, some of the heat supplied to the metal rod is conducted to and carried away ‘yy the surrounding air. If the heat supply to the rod is interrupted, heat wil continue to be carried away from the rod by the air surrounding until the tempera- ture of the rod drops to that of the air. When this occurs, there will be no temperature differ- ‘ential, the system will be in equilibrium, and no heat will be transferred. ‘The rate of heat transfer by conduction, a8 previously stated, is in direct proportion to the difference in temperature between the high and low temperature parts. However, all materials do not conduct heat at the same rate. Some materials, such as metals, conduct heat very readily, whereas others, uch as glass, wood, and cork, offer considerable resistance to the con- duction of heat. Therefore, for any given tem- perature difference, the rate of heat flow by conduction through different materials of the ‘same length and cross section will vary with the particular ability of the various materials to conduct heat. The relative capacity of a material to conduct heat is known as its conductivity. Materials which are good conductors of heat ‘have a high conductivity, whereas materials which are poor conductors. have a low con- ductivity and are used as heat insulators. In general, solids are better conductors of heat than liquids, and liquids are better con- ductors than gases. This is accounted for by the difference in the molecular structure. Since the molecules of a gas are widely separated, the transfer of heat by conduction, that is, from molecule to molecule, is difficult. 2,20, Convection. Heat transfer by con- ‘ection occurs when heat moves from one place to another by means of currents which are set up within some fiuid medium, These currents are known as convection currents and result from the change in density which is brought about by the expansion of the heated portion of the fluid, ‘cooler portions of water descend to Feplace the lighter portions that ise Heated portions of water become lighter and rise toward surface, thereby distributing the heat throughout the entire mass Fig. 2-2. Convection currents set up in a vessel of water when the vessel i heated at bottom center. When any portion of a fluid is heated, it ex- pands and its volume per unit of weight increases. Thus, the heated portion becomes lighter, rises to the top, and is immediately replaced by a cooler, heavier portion of the fluid. For example, assume that a tank of ‘water is heated on the bottom at the center Fig. 2-2). The heat from the flame is conducted ‘through the metal bottom of the tank to the water inside. As the water adjacent to the heat source absorbs heat, its temperature increases and it expands. ‘The heated portion of the ‘water, being lighter than the water surrounding, rises to the top and is replaced by cooler, more dense water pushing in from the sides. As this new portion of water becomes heated, it too rises to the top and is replaced by cooler water from the sides. As this sequence continues, the heat is distributed throughout the entire mass of the water by means of the convection currents, established within the mass. ‘Warm air currents, such as those which occur over stoves and other hot bodies, are familiar to . How convection currents are utilized to carry heat to all parts of a heated space is ‘ustrated in Fig, 2 2-21, Radiation. Heat transfer by radiation ‘occurs in the form of a wave motion similar to tight waves wherein the energy is transmitted from one body to another without the need for 16 PRINCIPLES OF REFRIGERATION Fig. 2-3. Room heated by natural convection, intervening matter. Heat energy transmitted by wave motion is called radiant energy. tis assumed that the molecules of a body are in rapid vibration and that this vibration sets up a wave motion in the ether surrounding the body.* Thus, the internal molecular energy of the body is converted into radiant energy waves. ‘When these energy waves are intercepted by ‘another body of matter, they are absorbed by that body and are converted into its internal ‘molecular energy. The earth receives heat from the sun by radiation, ‘The energy of the sun's molecular vibration is imparted in the form of radiant ‘energy waves to the ether of interstellarspacesur- rounding the sun. The energy waves travel across billions of miles of space and impress their ‘energy upon the earth and upon any other material bodies which intercept their path. The radiant energy is absorbed and transformed into internal molecular energy, so that the vibratory motion of the hot body (the sun) is ‘reproduced in the cooler body (the earth). All materials give off and absorb heat in the form of radiant energy. Any time the tempera- ture of a body is greater than that of its sur- Toundings, it will give off more heat by radiation than it absorbs. “Therefore, it loses energy to its surroundings and its internal energy de- creases, Ifthe temperature of the body is below that ofits surroundings, it absorbs more radiant ‘energy than it loses and its internal increases. When no temperature difference ‘Ether is the name given to that which fills all space unoccupied by matter, such as interstelar space and the space between the molecules of every ‘material, exists, the energy exchange is in equilibrium and the body neither gains nor loses energy. Heat transfer through a vacuum is impossible by either conduction or convection, since these processes by their very nature require that matter be the transmitting media. Radiant energy, on the other hand, is not dependent ‘upon matter as a medium of transfer and there fore can be transmitted through a vacuum. Furthermore, when radiant energy is transferred from a hot body to a cold body through some intervening media such as air the temperature of the intervening media is unaffected by the passage of the radiant energy. For example, ‘heat is radiated from a “warm’ wall to a “cold” wall through the intervening air without having any appreciable effect upon the temperature of the air. Since the molecules of the air are rela- tively few and widely separated, the waves of radiant energy can easily pass between them so that only a very small part of the radiant energy is intercepted and absorbed by the molecules of the air. By far the greater portion of the radiant. ‘energy impinges upon and is absorbed by the solid wall whose molecular structure is much ‘more compact and substantial. Heat waves are very similar to light waves, 4iffering from them only inlength and frequency. Light waves are radiant energy waves of such length as to be visible to the human eye. Thus, light waves are visible heat waves. Whether hheat waves are visible or invisible depends upon the temperature of the radiating body. For example, when metal is heated to a sufficiently high temperature, it will “glow,” that is, emit visible heat waves (light). When radiant energy waves, cither visible or invisible, strike a material body, they may be reflected, refracted, or absorbed by it, or they ‘may pass through it to some other substance beyond. ‘The amount of radiant energy which will pass through a material depends upon the degree of transparency. A highly transparent material, soc car gla owl ow mow ef th radiant energy to pass through to the materials beyond, whereas opaque materials, such as wood, metal, cork, etc., cannot be penetrated by radiant energy waves and none will pass through ‘The amount of radiant energy which is either reflected or absorbed by a material depends MATTER, INTERNAL ENERGY, HEAT, TEMPERATURE 17 upon the nature of the material’s surface, that is, its texture and its color. Materials having a light-colored, highly polished surface, such as a mirror, reflect a maximum of radiant energy, ‘whereas materials having rough, dull, dark surfaces will absorb the maximum amount of radiant energy. 222. British Thermal Unit. It has already ‘been established that a thermometer measures ‘only the intensity of heat and not the quantity. However, in working with heat it is often necessary to determine heat quantities. Obvi- }, some unit of heat measure is required. Heat a form of ency, and ooh intangible and cannot be measured directly. Heat can be measured only by the ‘effects it has on a material, such as the change in temperature, state, color, size, ete. ‘Themost universally used unit of heat measure is the British thermal unit, abbreviated Bru. A ‘Btuis defined as the quantity of heat required to cchange the temperature of 1 Ib of water 1°F. ‘This quantity of heat, if added to 1 Ib of water, will raise the temperature of the water 1°F. Likewise, if 1 Btu is removed from 1 1b of water, the temperature of the water will be lowered Pr ‘The quantity of heat required to change the temperature of 1 1b of water 1° F is not a con- ‘stant amount. It varies slightly with the tem- perature range at which the change occurs. For this reason, a Btu is more accurately defined as, ‘being 1/180th of the quantity of heat required to raise the of 1 Ib of water from the freezing point (32”F) to the boiling point @12° F). ‘This is identified as the “mean Btu"* and is the exact amount of heat required to raise the temperature of 1 Ib of water from 62 to 63°F. If the change in temperature occurs at any other point on the temperature scale, the amount of heat involved is either more or less than the mean Btu, depending upon the par- ticular point on the temperature scale that the ‘change takes place. However, the variation from the mean Btu is so slight that it may be neglected and, regardless of the temperature range, for all practical purposes itis sufficiently ‘accurate to assume that the temperature of 1 1b of water is changed 1°F by the addition or removal of 1 Btu. 223, Specific Heat. ‘The specific heat of a ‘material is the quantity of heat required to change the temperature of 1 pound of the material 1° F. For instance, the specific heat of aluminum is 0.226 Btu/lb/"F, whereas that.of brass is 0.089 Btu/Ib/"F. This means that 0.226 Btu is required to raise the temperature of 1 pound of aluminum 1° F, whereas only 0.089 Btu is necessary to change the temperature of 1 pound of brass 1° F. Note that by the defini- tion of the Btu the specific heat of water is 1 [Btu per pound per degree Fahrenheit. ‘The specific heat of any material, like that of water, varies somewhat throughout the tem- perature scale. Here again, the variation is so slight that it is sufficiently accurate for most calculations to consider the specific heat to be ‘4 constant amount. This is not true, however, as the material ‘through a change in physical state. The specific heat of a material in the solid state is approximately one-half that of the same-material in the liquid state, For instance, the specific heat of ice is 0.5 Buu, whereas that of water is one. The specific heat values of materials in the gaseous state are dis- cussed in another chapter. 2-24, Calculating Heat Quantity. ‘The quantity of heat which must be added to or removed from any given mass of material in order to bring about a specified change in its temperature can be computed by using the following equation: Q, = My - 4) 8) where Q, = the quantity of heat either absorbed or rejected by the material ‘M =the weight of the material in pounds C = the specific heat of the material 4 = the initial temperature 4 = the final temperature Example 28, Twenty of water at an initial temperature of 76° F are heated until the is increased to 180° F. How ‘much heat must be supplied? meine” 20 Ib (180 apa = 2016 x1 x (90 = 1 Bios 24, © Sow ea ° Example 2.9, If water weighs 8.33 Ib per ‘how much heat is rejected by 30 gal of ‘water in cooling from 80° F to 35° F? 18 PRINCIPLES OF REFRIGERATION Solution. Weight of water in 30 gal x 8.33 Ib/gal pounds = 2501 Applying, Qs = 2501b x 1 x (35 = 80) ion 28," = 2501b x 1 x (—45 Equation = 11,250 Btu Nore: Since the specific heat of a material is given in terms of Btu/lbj° F, the weight of the material must be determined before Equation 2-8 can be applied. less than f,, the answer obtained appl uation 2-8 will be negative, iit that Heat i rpeted by rates tha absorbed” by the material. In this type of problem, where the direction of heat flow is ‘obvious, the negative sign can be ignored and the answer assumed to be positive. Example 2-10. Fifteen pounds of cast iron are cooled from 500° F to 250° F by being im- mersed in 3 gallons (25 Ib) of water whose initial temperature is 78° F. Assuming that the specific heat of the cast iron is 0.101 Btu/lb/° F and that all of the heat given up by the cast iron is absorbed by the water, what is the final ‘temperature of the water? Solution. By applying Equation 2-8 to compute the total quantity ofheat —Q, = 15 1b x 0.101 given up by the cast x 250 iron, = 378.75 Btu rearranging and 2. applying Equation2-8 4 “370 +4 to determine the final 7.75 ‘temperature of the BA +r water after absorbing 25* the heat given up by oss 4 10F the cast iron, =93°F 2.25. Heat Divided into Two Kinds or Categories. It has been previously stated (Section 2-6) that heat has the ability to bring about a change in the physical state of a material as well as the ability to cause a change in its temperature, Heat is divided into two kinds or categories, depending upon which of these two effecs it has on a material which either absorbs fr rejects it. ‘The division of heat into several classifications is made only to facilitate and simplify certain necessary calculations and does not stem from any difference in the nature of heat itself. 2-26. Sensible Heat. When heat either ab- sorbed or rejected by a material causes or accompanies a change in the temperature of the material, the heat transferred is identified as sensible heat. The term sensible is applied to this particular heat because the change in tem- perature it causes can be detected with the sense Of touch and can, of course, be measured with a thermometer. 2:27, Latent Heat. When heat, either added to of rejected by a material, brings about or accompanies a change in the physical state of the material, the heat is known as latent heat. The name latent, a Latin word meaning hidden, is said to have been given to this special kind of, heat by Dr. Joseph Black because it apparently disappeared into a material without having any effect on the temperature of the material. ‘Many materials progressing up the tempera~ ture scale will pass through two changes in the state of aggregation: first, from the solid to the liquid phase and then, as the temperature of the liquid is further increased to a certain level beyond which it cannot exist as a liquid, the lguid wil change into the vapor state, When ‘occurs in either direction between thesolldsnd liquid phases, the heat involved is known as the latent heat of fusion. When the change occurs between the liquid and vapor pphases, the heat involved is the latent heat of vaporization. 2-28. Sensible Heat of a Solid. To obtain a ‘better understanding ofthe concept of molecular ‘energy, consider the progressive effects of heat as it is taken in by a material whose initial thermodynamic condition is such that its energy content is zero. Assume that a solid in an open container is at a temperature of —460°F (Absolute Zero). Theoretically, at this tem- perature the molecules of the material have no energy and are completely at rest. ‘When heat energy flows into the solid, the ‘molecules of the solid begin to move slowly and the temperature of the solid begins to climb. ‘The more heat energy taken in by the solid, the. faster the molecules vibrate and the warmer the solid becomes. The increase in molecular velo- city and in the temperature of the solid continues as more heat is absorbed, until the solid reaches its melting or fusion temperature. The total quantity of heat energy required to bring the ‘temperature of the solid from the original con- dition of Absolute Zero to the melting or fusion ‘temperature is known as the sensible heat of the MATTER, INTERNAL ENERGY, HEAT, TEMPERATURE 19 solid. As previously shown, the quantity of heat which must be transferred in order to bring about a specified change in the temperature of any given mass of any material can be calculated by applying Equation 2-8. 2:29. The Melting or Fusion Temperature. Upon reaching the fusion temperature, the molecules of the solid are moving as rapidly as {is possible within the rigid molecular structure of the solid state. It is not possible to increase further the motion of the molecules or the tem- perature of the solid beyond this point without first overcoming partially the forces of mutual attraction which exists between the molecules. Hence, the material cannot exist in the solid state at any temperature above its melting or fusion temperature. On reaching the fusion ‘temperature, any additional heat absorbed by ‘the material will cause some part of the solid to revert to the liquid phase. ‘The exact temperature at which melting or fusion occurs varies with the different materials and with the pressure. For instance, at normal ‘atmospheric pressure, the fusion temperature of lead is approximately 600° F, whereas copper melts at approximately 2000° F and ice at only 32°F. In general, the melting temperature decreases as the pressure increases except for nonerystalline solids, whose melting tempera~ tures increase as the pressure increases. 2-30. Latent Heat of Fusion. When heat is absorbed by a solid at the fusion temperature, the molecules of the solid utilize the energy to overcome partially their attraction for one another. They break away from one another to some extent and become more widely separated. ‘As the molecules flow over and about one another, the material loses the rigidity of the solid state and becomes fluid, It can no longer support itself independently and will assume the shape of any containing vessel. ‘The attraction which exists between the mole- cules of a solid is considerable and a relatively large quantity of energy is required to overcome that attraction. The quantity of heat required ‘to melt one pound of a material from the solid pphase into the liquid phase is called the latent heat of fusion. ‘The latent heat of fusion, along with other values such as specific heat, fusion temperature, eic., for the different materials has ‘been determined by experiment and may be found in various tables. tis important at this point to emphasize that the change of phase occurs in either direction at the fusion temperature, that is, the temperature t which the solid will melt into the liquid phase is the same as that at which the liquid will freeze into the solid phase. Further, the quan- tity of heat that must be rejected by a certain ‘weight of liquid at the fusion temperature in order to freeze into the solid state is exactly equal to the amount of heat that must be: ab- sorbed by the same weight of the solid in melting into the liquid state. ‘None of the heat absorbed or rejected during the change of phase has any effect on molecular velocity. Therefore the temperature of the ‘material remains constant during the phase change, and the temperature of the resulting liquid or solid is the same as the fusion tem- perature.* ‘The quantity of heat that is absorbed by & given weight of a solid at the fusion temperature in melting into the liquid phase, or, conversely, the quantity of heat that is rejected by a given ‘weight of liquid at the fusion temperature in freezing or solidifying, can be determined by applying the following equation: On =M x hy where Qz, = the quantity of heat in Btu ‘M = the mass or weight in pounds hy = the latent heat in Btu per pound Example 2-11. Calculate the quantity of heat required to melt 12 Ib of ice at 32° F into water at 32°F, The latent heat of fusion of water under atmospheric pressure is 144 Btu per pound. Solution. Apply ing Equation 2, the quantity of heat re- quired to melt 121b 12 1b x 144 Btu/lb of ice 1728 Btu Nore. Since 12 Ib of ice absorb 1728 Btu in ‘melting into water, it follows that 12 Ib of water 2° F will reject 1728 Btu in returning to the solid state. @9) * This applies with absolute accuracy only to crystalline solids. Noncrystalline solids, such as glass, have indefinite fusion temperatures. That is, the temperature will vary during the change of phase. However, for the purpose of calculating heat ‘quantities, the temperature is assumed to remain ‘constant during the phase change 20 PRINCIPLES OF REFRIGERATION Example 2:12. If 50 1b of ice at 32°F absorb 6000 Btu, what part of the ice will be ‘melted? iin, By earngng = r ation 3 the pet of te i mcd M8 ‘144 Bujlb = 41.66 1b 231, Sensible Heat of the Liquid. When a material passes from the solid to the liquid phase, the resulting liquid is at the fusion tem- perature. The temperature of the liquid may then be increased by the addition of heat, Any heat absorbed by a liquid after the change of state is set up in the liquid as an increase in the internal kinetic energy. Molecular velocity increases and the temperature of the liquid rises. But here again, as in the case of the solid, the ‘temperature of the liquid eventually reaches a point beyond which it cannot be further in- creased. A liquid cannot exist as a liquid at any ‘temperature above its vaporizing temperature for a given pressure and, upon the vaporizing temperature, if additional heat is taken in by the liquid, some part of the liquid will change to the vapor phase. ‘The total quantity of heat taken in by a liquid as its temperature is increased from the fusion to the vaporizing temperature is called the sensible heat ofthe liquid. Here again, Equation 2-8, sometimes known as the “sensible heat equation,” can be applied to determine the quantity of heat necessary to change the tem- perature of any given weight of liquid through any specified temperature range. 232 Saturation Temperature. The tem- perature at which a liquid will change into the vapor phase is called the saturation temperature, sometimes referred to as the “boiling point or “boiling temperature.” A liquid whose tem- perature has been raised to the saturation tem- perature is called a saturated liquid. ‘The saturation temperature, that is, the tem- erature at which vaporization occurs, is different for each liquid. Iron, for example, vaporizes at 4450°F, copper at 4250" F, and lead 3000° F. Water, of course, boils at 212° F, and alcohol at 170°F. Some liquids boil at extremely low temperatures. A few of these are ammonia, oxygen, and helium, which boil at temperatures of 28°F, —295°F, and 452° F, respectively. 2.33, Latent Heat of Vaporization. Any heat taken in by a liquid after the liquid reaches the saturation temperature is utilized to increase the degree of molecular separation (increases the {internal potential energy) and the substance changes from the liquid to the vapor phase.* ‘There is no increase in molecular velocity and, therefore, no change in the ‘internal kinetic energy during the change in phase. Hence, the temperature remains constant during the ‘phase change and the vapor which results is at the vaporizing temperature. AAs the material changes state from a liquid toa vapor, the molecules of the material acquire sufficient energy to overcome all restraining forces, including the force of gravity. The amount of energy required to do the internal work necessary to overcome these restraining forcesis very great. For this reason, the capacity of a material to absorb heat while undergoing 1a change from the liquid to the vapor phase is, enormous, many times greater even than its ‘capacity to absorb heat in changing from the solid to the liquid phase. ‘The quantity of heat which 1 tb of a liquid absorbs while changing into the vapor state is known as the latent heat of vaporization. The latent heat of vaporization, like the saturation temperature, is different for each material. Tt will be shown later that both the latent heat value and the saturation temperature of any particular liquid vary with the pressure over the liquid. When the pressure increases, the satur- ation temperature increases and the latent heat value decreases. ‘The quantity of heat required to vaporize any given weight of liquid at the saturation tem- OL=M x hy where Qz, = the quantity of heat in Btu ‘M = the mass or weight-in pounds ‘yy = the latent heat of vaporization in Beu/lb Example 2-13. If the latent heat of vapori- zation of water is 970 Btu per pound, how * Some of the energy added to the material leaves ‘the material as external work and has no effect on the internal energy of the material. When the pressure is constant, the amount of extemal work ‘done is proportional to the change in volume. External work is discused in detail later. MATTER, INTERNAL ENERGY, HEAT, TEMPERATURE 21 ‘much heat is requited to vaporize 3 gal of water at the saturation temperature of 212° F? Solution. Total 3 gal x 8.33 Ibjgal weight of water M = =251b ApplyingEquation _—25 Ib x 970 Btu/lb 2-10, Oy, = 24,250 Btu Example 214, One gallon of water at 200° F in an open container absorbs 1200 Btu. How much water is vaporized? Solution. Since the saturation temperature of water at atmospheric pressure is 212° F, the entire mass of the water must be raised to this ‘temperature before any water will vaporize Weight of I gal of water = 8.33 1b Applying Equation 2-8, the heat requited to raise the temperature of the 8.33 1b x 1 water from 200° F to x 12° 212°F, Q, = 100 Btu ‘Heat available to vapor- ize some portion of the 200 — 100 water = 1100 Btu Rearranging and apply: 1100 ing Equation 2-10, the 370 weight of water vaporized, = 1.135 1b or M 01136 gal Example 215. If 5000 Btu are removed from 8 Ib of saturated steam at atmospheric pressure, how much of the steam will condense into water? |, Solution. By rearrang- 4, _ 5000 Btu ing and applying Equation * 970 Braj 2-10, 5.15 Ib 2.34. Superheat—the Sensible Heat of a Vapor. Once a liquid has been vaporized, the temperature of the resulting vapor can be further increased by the addition of heat. The heat added to a vapor after vaporization is the sensible heat of the vapor, more commonly called supetheat. When the temperature of vapor has been so increased above the satur- ation temperature, the vapor is said to be supetheated and is called a superheated vapor. Superheated vapors are discussed at length in another chapter. 235. Total Heat. The total heat of a ‘material at any particular condition is the sum ‘otal ofall the sensible and latent heat required to bring it to that condition from an initial condition of Absolute Zero.* Example 2-16, Compute the total heat content of 1 Ib of steam at 212° F. Solution. The total heat of 1 tb of saturated steam is the sum of the following heat quantities: @ To raise the temperature of I Ib of ice from 460° F to 32° F, applying Equation Q, = 1 x 0.5 x zB 12 — (460) =P x05 e492 246 Bia (© To melt 1 tb of ice at 32° F into water at 32°F, applying Equation 2-9, Qy, = 1 x 144 = 144 Bm (© To increase temperature of water from 32° F to 212°F, applving Equation Q, = 1 x1 x s (212 — 32) =1x1 x 180 = 180 Btu @ To vaporize 1 Ib of water, applying Equation 2-10, (© Summation: O, =1 x 970 = 970 Btu Sensible heat of the solid = 246 Btu Latent heat of fusion = 144 Bu Sensible heat of the liquid = 180 Btu Latent heat of vaporization = 970 Btu Total heat of 1 Ib of steam = 1540 Bu ‘Through the use of a temperature-heat diagram, the solution to Example 2-15 is shown graphically in Fig. 2-4. 2-36, Mechanical Energy Equivalent. Normally the external energy of a body is ex- ‘in mechanical energy units (work), whereas the internal energy of a body is ex- pressed in heat energy units. The fact that internal energy is usually ex- Pressed in heat energy units gives rise to the definition of heat as molecular or internal energy. As previously stated, from a thermodynamic * The total heat of a material is commonly known as “enthalpy,” and is computed from some arbitrarily selected zero point rather than from ‘Absolute Zero. See Section 4-18, 22 PRINCIPLES OF REFRIGERATION ¥ g £ aii, foie | Hp ane vi ; §ss3 424258 4: ; yeep) ml Constant temperate wl 2 ok Constant temperature 4 2) + 7 n 4 pod ft Fg oa Heat content (Btu) FFig.2-4. Graphical analysis of the relationship of heat content to the temperature and state of 2 material, point of view energy is heat energy only when it is in transition from one body to another because of a difference in temperature between the two bodies. Once the energy flows into a body it becomes “stored” thermal energy. Hence, thermodynamically speaking, internal energy is not heat but thermal energy in storage. ‘Not all the heat energy flowing into a body is stored in the body as internal energy. In many instances, some or all of the energy flowing into the body passes through or leaves the body 28 ‘work (mechanical energy). This is made clear in another section. Furthermore, up to this point it has been assumed that the intemal energy of a body is, {increased only by the addition of heat energy directly, as from a flame or some other heat source.’ However, this is not the case. The internal or molecular energy of a body may also bbe increased when work is done on the body. ‘That is, the mechanical energy of the work done ‘on 8 body may be converted to the internal ‘nergy of the body. For example, the head of a nail struck by a hammer will become warm as a part of the mechanical energy of the hammer blow is converted to the internal kinetic energy of the nail head. As the molecules of the metal that make up the nail head are jarred and agi- tated by the blow of the hammer, their motion fr velocity is increased and the temperature of the nail head increases. Ifa wire is bent rapidly back and forth, the bent portion of the wire becomes hot because of the agitation of the ‘molecules. Also, everyone is familiar with the increase in temperature which is brought about, by the friction of two surfaces rubbing together. Often the external energy of a body is con- verted to internal energy and vice versa. For example, a bullet speeding toward a target has Kinetic energy because of its mass and velocity. ‘At the time of impact with the target, the bullet loses its velocity and a part of its kinetic energy is imparted to the molecules of both the bullet and the target so that the internal energy of cach is increased. Since heat energy is often converted into mechanical energy (work) and vice versa, and since it is often desirable to express both the MATTER, INTERNAL ENERGY, HEAT, TEMPERATURE 23 internal and external energies of a body in terms of the same energy unit, a factor which can be used to conyert from one energy unit to the other is useful. Tt has been determined by experiment that one Btu of heat energy is equivalent to 778 ft-lb of mechanical energy, that is, one Btu is the amount of heat energy required to do 778 ft-Ib of work, This quantity is known as the mecha- nical energy equivalent and is usually repre- sented in equations by the symbol J. To convert energy in Btu into energy in foot-pounds, the energy in Btu is multiplied by 718 and, to convert energy in foot-pounds to energy in Btu, the energy in foot-pounds is divided by 778. Expressed as equations, these relationships become w e-F Cony and Weoxs @-12) where Q = the quantity of heat energy in Btu W = mechanical energy or work in foot- pounds JJ = the mechanical energy equivalent of beat Example 217. Convert 36,000 ftlb of ‘mechanical energy into heat energy units. i . 36,000 i Srltion. Applying Equa Q = “77q~ tion , = 46.3 Btu Example 2-18, Express 12 Btu of heat energy as work in mechanical energy units. Worx 78 = 9336 fllb Solution. Equation 2-12, PROBLEMS 1. A Fahrenheit thermometer reads 85°. What 4s the temperature in degrees Centigrade? ‘Ans, 29.44° C 2, Convert 90° Centigrade to degrees Fahren- heit. Ans, 194° F 3. The ture of a gas is 40°F. What is its temperature on the Rankine scale? ‘Ans. 500° R 4. The temperature of the suction vapor enter- ing a refrigeration compressor is -20° F. What is the temperature of the vapor in degrees Rankine? Ans. 440" R. 5. of water are heated from 75° F to 180°F, Determine the quantity of heat required? “Ans. 36240 Bt 6. In a certain industrial process, 5000 gal of ‘water are cooled from 90° F to 55° F each hdur. Determine the quantity of heat which must be removed each hour to produce the required cooling. ‘Ans. 1,457,750 Btu 7. Calculate the quantity of heat which must be removed from 60 gal of water in order to cool the water from 42°F and freeze it into ice at 32°F. Ans. 77,000 Btu 8. If 12,120 Btu are added to 3 gal of water at 200° F, what fraction of the water in will be vaporized? Ans. 9.4 1b or 1.13 gal 9. Twenty-five pounds of ice are placed in 10 gal of water and allowed to melt. that there is no loss of heat to the surroundings, if the initial temperature of the water is 80°F, to what temperature will the water be cooled by the melting of the ice? Ans. 35.7° 10.4 ding in a cylinder does 25,000 fib of work on the piston. Determine the ‘quantity of heat required to do the work. ‘Ans, 32.13 Btu 3 Thermodynamic Processes 3:1. The Effects of Heat on Volume. When either the velocity of the molecules or the degree of molecular separation is increased by the ‘addition of heat, the mean distance between the ‘molecules is increased and the material expands so that a unit weight of the material occupies a ‘greater volume. This effect is in strict accord- ance with the theory of increased or decreased molecular activity as described earlier. Hence, ‘when heat is added to or removed from an ‘unconfined material in any of the three physical states, it will expand or contract, respectively. ‘Thats, its volume will increase or decrease with the addition or removal of heat. (One of the few exceptions to this rule is water. If water is cooled, its volume will decrease ‘normally until the’ temperature of the water drops to 39.2°F, At this point, water attains its maximum density and, if further cooled, its volume will again increase, Furthermore, after being cooled to 32°F, it will solidify and the solidification will be accompanied by still further expansion. In fact, 1 cu ft ot water wl freeze into approximately 1.085 cu ft of ice. This accounts for the tremendous expansive force created during solidification which is sufficient to burst steel pipes or other restraining vessels, ‘The peculiar behavior of water as it solidifies ‘appears to contradict the general laws govern- ing molecular activity as described previously. However, this is not the case. The unusual ‘behavior of water is explained by the hypothesis, that, although the molecules of water are actually closer together in the solid state than they are in the liquid state, they are grouped together to form crystals. It is the relatively large spaces between the crystals of the solid, rather than any increase in the mean distance between the molecules, which accounts for the ‘unusual increase in volume during solidification. This is true also for crystalline solids other than ice, 32. Expansion of Solids and Liquids. When a solid or a liquid is heated so that its tempera- ture is increased, it will expand a given amount for each degree of temperature rise. As stated earlier, many temperature measuring devices are based upon this principle. The amount of expansion which a material experiences with each degree of temperature rise is known as its coefficient of expansion. The coefficient of expansion is different for every material, and ‘moreover it will vary for any particular material depending upon the temperature range in ‘which the change occurs. Since solids and liquids are not readily com- Pressible, if a solid or a liquid is restrained or ‘confined so that its volume is not allowed to cchange normally with a change in temperature, ‘tremendous pressures are created within the ‘material itself and upon the restraining bodies, which is likely to cause buckling or rupturing of either the materia, the restraining bodies, or both. To provide for the normal expansion and contraction occurring with temperaturechanges, expansion joints are built into highways, bridges, pipelines, etc. Likewise, liquid con- tainers are never completely filled. Space must bbe allowed for the normal expansion, Other- wise the tremendous expansive forces generated by a temperature increase will cause the con- taining vessel to rupture, sometimes with explosive force. 33. Specific Volume. The specific volume of a material is the volume occupied by a 1 Ib mass of the material. Each material has a different specific volume and, because of the change in volume which accompanies a change in temperature, the specific volume of every ‘material varies somewhat with the temperature range. For instance, at 40° F, 1 Ib of water has a specific volume of 0.01602 cu ft, whereas the volume occupied by 1 Ib of water at 80° F is 0.01608 cu ft. 3-4, Density. The density of a material is the ‘weight in pounds of 1 cu ft of the material. Density is the reciprocal of specific volume, that is, the specific volume divided into one. The density of any material, like specific volume, ‘varies with the temperature, but in the opposite direction. For example, at 40° F, the density of water is 62.434 Ib per cubic foot (1/0.01602), whereas water at 80° F has a density of 62.20 I per cubic foot (1/0.01608). Since density and specific volume are reciprocals of each other, a5 one increases the other decreases. The density and/or the specific volume of many ‘common materials can be found in various tables. ‘The relationship between density and specific volume is given by the following equations: pat on 1 ont G2 VeaMxo @3) M=Vxp oH Where » = the specific volume in cubic feet per pound (cu fy/Ib) p= the density in pounds per cubic foot jeu £2) ¥ = the total volume in cubic feet ‘M = the total weight in pounds Example 31. If the specific volume of dry saturated steam at 212° F is 26.80 cu ft per pound, what is the density of the steam? Solution. Ay - Equation 3-1, sm = 0.0373 tbjeu ft density p Exemple 32. The basin of a cooling tower, Sift x 4ft x 1 fh is filled with water. Af the density of the water is 26.8 Ib per cubic foot what ste tol weight of the water in te Solution. The total volume V Sift x 4ft xift =cut Applying Equation 3-4, the 20 x 26.8 total weight of water M1 = 536 Ib THERMODYNAMIC PROCESSES 25 3.5, Pressure-Temperature-Volume Rela- tionships of Gases. Because of its loose ‘molecular structure, the change in the volume of 8 gas as the gas is heated or cooled is much ‘greater than that which occurs in the case of a solid of a liquid, In the following sections, it will be shown that a gas may change its con- ition in a number of different ways and that certain laws have been formulated which govern the relationship between the pressure, temper- ature, and volume of the gas during these changes. It should be noted at the outset that in applying the fundamental gas laws it is always necessary to use absolute pressures and absolute temperatures in degrees Rankine. Further, in studying the following sections, it should be remembered that a gas always ‘completely fills any container. ‘The relationship between the pressure, temperature, and volume of a gas is more ‘easily understood when considered through a series of processes in which the gas passes from some initial condition to some final con- dition in such a way that only two of these properties vary during any one process, whereas the third property remains unchanged or con- stant, 34, Temperature-Volume Relationship at Constant Pressure. If a gas is heated under such conditions that its pressure is kept constant, its volume will increase 1/492 of its volume at 32°F for each 1° F increase in its ‘temperature. Likewise, if a gas is cooled at a ‘constant pressure, its volume will decrease 1/492 of its volume at 32° F for each 1° F decrease in its temperature. In order to better visualize a constant pressure change in condition, assume that a gas is con- foa'in a clinter eqeipped with « precy fitting, frictionless piston (Fig. 3. row ofthe gus at whish exert on the {gas by the weight ofthe piston and by the weight ‘of the atmosphere on top of the piston. Since the piston is free to move up or down in the cylinder, the gas is allowed to expand or con- tract, that is, to change its volume in such a way that the pressure of the gas remains constant. As the gasis heated, its temperature and volume increase and the piston moves upward in the cylinder. As the gas is cooled, its temperature ‘and volume decrease and the piston moves down- ward in the cylinder, In either case, the pressure 26 PRINCIPLES OF REFRIGERATION added @ o Heat removed © Fig. #1. Constant pressure process. (o) Gas confined in a cylinder with a perfectly fitting, frictionless piston. (8) AS gas Is heated, both the temperature and the volume of the gas Increase. The increase in volume ‘exactly proportional to the increate in absolute temperature. (c) As gas Is cooled, both the temperature and ‘the volume of the gas decrease, The decreate in volume Is exactly proportional to the decrease in absolute ‘temperature. of the gas remains the same or unchanged during the heating or cooling processes. 3-7. Charles’ Law for a Constant Pressure Process. Charles’ law for a constant pressure ‘process states in effect that, when the pressure Of the gas remains constant, the volume of the gas varies directly with its absolute temperature. Thus, if the absolute temperature of a gas is doubled while its pressure is kept constant, its volume will also be doubled. Likewise, if the absolute temperature of a gas is reduced by one-half while the pressure is kept constant, its volume will also be reduced by one-half. This relationship is illustrated in Figs. 3-16 and Be. Charles’ law for a constant pressure process written as an equation is as follows: When the pressure is kept constant, TWiT os where 7, = the initial temperature of the gas in degrees Rankine Ty = the final temperature of the gas in Rankine Vy =the initial volume of the gas in cubic feet Vq = the final volume of the gas in cubic feet ‘When any three of the preceding values are known, thefourth may be calculated by applying Equation 3-5. Example 33. A gas, whose initial tem- perature is 520° R and whose initial volume is 5 cu ff, is allowed to expand at a constant pressure until its volume is 10 cu ft. Determine the final temperature of the gas in degrees Rankine. ‘Solution. By ng and applying Equation 3-5, the final temperature of the gas 7, Example 34. A gas, having an initial temperature of 80°F, is cooled at a constant pressure until its temperature is 40°F. If the Initial volume of the gas is 8 cu ft, what is its final volume? Solution. Since the temperatures are given in degrees Fahrenheit, they must be converted to degrees Rankine before being substituted in Equation 3-5. By rearranging and apply. _ Ta¥ ing ‘equation 355, the final T; volume Vs 500 x8 340 = 74074 ca ft 34, Pressure-Volume Relationship at a Constant Temperature. When the volume of a gas is increased or decreased under such ‘conditions that the temperature of the gas does mot change, the absolute pressure will vary inversely with the volume. Thus, when a gas is ‘compressed (volume decreased) while its tem- perature remains unchanged, its absolute Pressure will increase in proportion to the decrease in volume. Similarly, when a gas is expanded at a constant temperature, its abso- ute pressure will decrease in proportion to the increase in volume, This is a statement of Boyle's law for a constant temperature process and is illustrated in Figs. 3-2a, 3-26, and 3-2c. It has been previously stated that the mole- cules of a gas are flying about at random and at high velocities and that the molecules of the {85 frequently collide with one another and with. the walls of the container. The exerted by the gas is a manifestation of these molecular collisions. Billions and billions of gas mole- cules, traveling at high velocities, strike the walls of the container during each fraction of a second. It is this incessant molecular bombard- ment which produces the pressure that a gas exerts upon the walls of its container. The ‘magnitude of the pressure exerted depends upon the force and frequency of the molecular im- pacts upon a given area. The greater the force Fig. 3:2. Constant tempera: ‘ture process. (a) Initial condi- ton. (2) Constant temperature xxpansion—volume change is Inversely proportional to the change in absolute pressure Heat must be added during expansion to keep tempera: ture constant. (e) Constant ‘temperature compression— volume change Is Inversely proportional to the change in absolute pressure. Heat must be removed during com- pression to keep temperature @ THERMODYNAMIC PROCESSES 27 and frequency of the impacts, the greater is the pressure. The number of molecules confined in 8 given space and their velocity will, of course, determine the force and the frequency of the impacts, That is, the greater the number of ‘molecules (the greater the quantity of gas) and the higher the velocity of the molecules (the hhigher the temperature of the gas), the greater is the pressure. ‘The force with which the mole- cules strike the container walls depends only upon the velocity of the molecules. ‘The higher the velocity the greater is the force of impact. The greater the number of molecules in a given, space and the higher the velocity the more often the molecules will strike the walls. ‘When a gas is compressed at a constant temperature, the velocity of the molecules re- mains unchanged. The increase in which occurs is accounted for by the fact that the volume of the gas is diminished and a given ‘number of gus molecules are confinedina smaller space so that the frequency of impact is greater. The reverse of this holds true, of course, when. the gas is expanded at a constant temperature. ‘Any thermodynamic process which occurs in. such a way that the temperature of the working ‘substance does not change during the process is called an isothermal (constant temperature) process, ‘Boyle's law for a constant temperature process 28 PRINCIPLES OF REFRIGERATION vamecian Pressure = 50 psia ‘Temperature = 250°R proportion to the increase in absolute temperature. (c) to the decrease in absolute temperature. is represented by the following equation: if ‘the temperature is constant, PY = PM 66 where P, = the inital absolute pressure ‘Py = the final absolute pressure V,, = the initial volume in cubic feet Va = the final volume in cubic feet Example 35. Five pounds of air are ex- inded at a constant temperature from an Initial volume of 5 cu ft to a final volume of 10 cu ft. If the initial pressure of the air is 20 psia, what is the final pressure in psia? Solution. and LAXK appying Beton 6h ial pressure Py Example 3-6, Four cubic fect of gas are allowed to expand at a constant temperature from an initial pressure of 1500 psfa to a final Frese of 900 pets Determine the Sal Volume of the gas. ‘Solution. By rearrangingand = _ Pav lator Dyrarrpeens =e "2 an Example 3-7. A given weight of gus, whose initial volume is 10 cu ft, is thermally until ts volume is 4'cu fIt the wu () The absolute pressure increases In direct sbtolute pressure decreases in direct proportion initial pressure of the gas is 3000 psfa, determine the final pressure in psig. ‘Solution. By rearranging and applying Equation 3-6, the ial pseu Py Dividing by 144 Subtracting the atmos- . pherie pressure = 37.38 psig 29, Pressure-Temperature Relationship ‘at a Constant Volume. Assume that a gas is confined in a closed cylinder so that its ‘volume cannot change as it is heated or cooled (Fig. 3-3a). When the temperature of a gas is increased by the addition of heat, the absolute pressure will increase in direct proportion to the increase in absolute temperature (Fig. 3-36). IF the gas is cooled, the absolute pressure of the gas will decrease in direct proportion to the decrease in absolute temperature (Fig. 3-3c). Whenever the temperature (velocity of the molecules) ofa gas is increased while the volume of the gas (space in which the molecules are confined) remains the same, the magnitude of the pressure (the force and frequency of mole- cular impacts on the cylinder walls) increases. Likewise, when a gas is cooled at a constant volume, the force and frequency of molecular impingement on the walls of the container diminish and the pressure of the gas willbe less than before. ‘The reduction in the force and the frequency of molecular impacts is accounted for by the reduction in molecular velocity. 3-10, Charles’ Law for a Constant Volume Process. Charles’ law states in effect that when a gas is heated or cooled under such con- ditions that the volume of the gas remains unchanged or constant, the absolute pressure varies directly with the absolute temperature. Charles’ law may be written as the following equation: when the volume is the same, on T, =the final temperature in degrees Rankine ‘P, = the initial pressure in pounds per ‘square inch absolute Py = the final pressure in pounds per square inch absolute Example 3-8. A certain weight of gas con- fined in a tank has an initial temperature of 80° F and an initial pressure of 30 psig. If the gas is heated until the final gage pressure is 30 psi, what is the final temperature in degrees Fahrenheit? aston By rearang neh ing and applying Equa- % tion 37, aan (80 + 460) 2 (00 +147) G0 + 14.7) = 12K Converting Rankineto 782 ~ 460 Fahrenheit 9 30F BIL. The General Gas Law. Combining ‘Charles’ and Boyle’s laws produces the following. equation: G8) Equation 3-8 isa statement that for any given weight of a gas, the product of the pressure in psfa and the volume in cubic feet divided by the absolute temperature in degrees Rankine will always be a constant. The constant, of course, will be different for different gases and, for any ‘one gas, will vary with the weight of gasinvolved. However, if, for any one gas, the weight of 1 Ib THERMODYNAMIC PROCESSES 29 is used, then ¥ will become the specific volume », and Equation 3-8 may be written: Po Po _R 7 os) where R = the gas constant ‘The gas constant R is different for each gas. ‘The gas constant for most common gases can bbe found in tables. A few of these are given in Table 3-1 ‘Multiplying both sides of Equation 3-9 by M produces Mv = MRT but since Mo=V then PV = MRT G-10) where P = the pressure in psfa V = the volume in ft? 'M = the mass in pounds R = the gas constant T = the temperature in ° R Equation 3-10 is known as the general gas law and is very useful in the solution of many problems involving gases. Since the value of R for most gases can be found in tables, ifany three of the four properties, P, V, M, and, areknown, the fourth property can be determined by Equa- tion 3-10. ‘Notice that the pressure must be in ‘pounds per square foot absolute. Example 3.9, The tank of an air compres- sor has a volume of 5 cu ft and is filled with air ata te of 100°F. If a gage on the tank reads 151.1 psi, what is the weight of the air in the tank? Solution. From Table 3-1, R for air = 53.3 By rearranging (si +147) and applying Equa- x 144x 5 tion 3-10, the weight ~ 33.3 x (100 + 460) of ait 1658 x 144 x5 353 x 500 = 4b Example 3-10. Two of air have a volume of 3 cu ft. If the pressure of the air is 135.3 psig, what is the temperature of the air in degrees Fahrenheit? 30 PRINCIPLES OF REFRIGERATION Solution. From Table 3-1, Rfor air = = 53.3 ‘rearranging talk and applying’ Equa- MR tion 3-10, the tem- (135.3 414.7) perature of the air _ 144 x3 in’R, 3K: 2150 x 144 x 3 Converting to Fahrenheit, 3-12, External Work. Whenever a material undergoes a change in volume, work is done. If the volume of the material increases,workis done by the material. If the volume of the material decreases, work is done on the material. For example, consider a certain weight of gas con- fined in'a cylinder equipped with a movable piston (Fig. 3-1a). As the gas is heated, its temperature increases and it expands, moving. the piston upward in the cylinder against the pressure of the atmosphere. Work is done in that the weight of the piston is moved through a distance (Fig, 3-16).* Theagency doing the work is the expanding gas. In order to do work, energy is required ection 1-12). In Fig. 3-16, the energy required to do the work is supplied to the gas as the gas is heated by an extemal source. It is possible, however, for a gas to do external work without the addition of energy from an external source. In such cases, the gas does the work at the expense of its own energy. That is, as the gas expands and does work, its internal kineticenergy (temperature) decreases in an amount equal to the amount of energy required to do the work. When a gas is compressed (its volume de- ‘reased), a certain amount of work must be done fon the gas in order to compress it. And, an amount of energy equal to the amount of work done will be imparted to the molecules of the gas during the compression. That is, the mechanical energy of the piston motion will be transformed into the internal kinetic energy of the gas (molecular motion) and, unfess the gas is cooled during the compression, the temperature of the gas will increase in proportion to the * Some work is done, alto, in overcoming friction and in overcoming the pressure of the atmosphere. amount of work done. The increase in the tem- perature of a gas as the gas is compressed is a ‘common phenomenon and may be noted by feeling the valve stem ofa tie being filled with a hand pump, or the head of an air compressor, etc. 3-13. The General Energy Equation. The aw of conservation of energy clearly indicates that the energy transferred to a body must be accounted for in its entirety. It has been shown, that some part (or al) of the energy taken in by a ‘material may leave the material as work, and that only that portion of the transferred energy which is not utilized to do external work remains in the body as “stored thermal energy.” It is evident then that all ofthe energy transferred to a body must be accounted for in some one or in some combination of the following three ways: (1) as an increase in the internal kinetic energy, @)asan increase in the internal potential energy, and (3) as external work done. The general energy equation is a mathematical statement of this concept and may be written: AQ=AK+AP +4" GA) where AQ = the heat energy transferred to the ‘material in Btu AK = that fraction of the transferred energy which increases the in- ternal kinetic energy AP = that fraction of the transferred ‘energy which increases the in- ternal potential energy ‘AW = that fraction of the transferred energy which is utilized 10 do external work The Greek letter, A (delta), used in front of a term in an equation identifies a change of condition. For instance, where K represents the internal kinetic energy, AK represents the ‘change in the internal kinetic energy. Depending upon the particular process or ‘change in condition that the material undergoes, any of the terms in Equation 3-11 may have any value either positive or negative, or any may be ‘equal to zero. This will be made clear later. 3-14. External Work of a Solid or Liquid. When heat added to a material in either the solid or liquid state increases the temperature of the material, the material expands somewhat and a small amount of work is done. However, the increase in volume and the external work done is so slight that the portion of the trans- ferred energy which is utilized to do extefnal work or to increase the internal potential energy is negligible. For all practical purposes, it can ‘be assumed that all the energy added to a solid for a liquid during a temperature change in- creases the internal kinetic energy. None leaves the material as work and none is set up as an increase in the internal potential energy. In this instance, both AP and AW of Equation 3-11 are equal to zero and, therefore, AQ is equal to AK. ‘When a solid melts into the liquid phase, the change in volume is again so slight that the external work done may be neglected. Further- more, since the temperature also remains constant during the phase change, none of the transferred energy increases the internal kinetic ‘energy. All the energy taken in by the melting solid is set up as an increase in the internal potential energy. ‘Therefore, AK and AW are ‘both equal to zero and AQ is equal to AP. ‘This is not true, however, when a liquid cchanges into the vapor phase. The change in volume that occurs and therefore the external work done as the liquid changes into a vapor is ‘considerable. For example, when 1 Ib of water fat atmospheric pressure changes into a vapor, its volume increases from 0.01671 cu ft to 26.79 ccu ft. Of the 970.4 Btu required to vaporize 1 Tb of water, approximately 72 Btu of this energy are required to do the work of expanding against the pressure of the atmosphere. The remainder of the energy is set up in the vapor as ‘an increase in the internal potential energy. In this instance, only AK is equal to zero, so that AQ is equal to AP plus AI. 315. “Ideal” or“Perfect” Gas. The various Jaws governing the pressure-volume-temperature relationships of gases as discussed in this chapter apply with absolute accuracy only to a hypo- thetical “ideal” or “perfect” gas. A “perfect” {88 is described as one in such a condition that there is no interaction between the molecules of the gas. The molecules of such a gas are entirely free and independent of each other's attractive forces. Hence, none of the energy ‘transferred either to or from an ideal gas has any effect on the internal potential energy. ‘The concept of an ideal or perfect gas greatly simplifies the solution of problems concerning THERMODYNAMIC PROCESSES 31 the changes in the condition of a gas. Many complex problems in mechanics are made simple by the assumption that no friction exists, the effects of friction being considered separately. ‘The function of an ideal gas isthe same as that of the frictionless surface. An ideal gas is assumed to undergo a change of condition without internal friction, that is, without the performance of internal work in the overcoming of internal molecular forces. ‘The idea of internal friction is not difficult to ‘comprehend. Consider that a liquid such as oil will not flow readily at low temperatures. This is because of the internal friction resulting from strong intermolecular forces within the liquid. ‘However, as the liquid is heated and the mole- ccules gain additional energy, the intermolecular forces are overcome somewhat, internal friction diminishes, and the liquid flows more easly. ‘Vaporization of the liquid, of course, causes fa greater separation of the molecules and brings about a substantial reduction in internal friction, but some interaction between the molecules of the vapor still exists. In the gaseous state, intermolecular forces are greatest when the gas is near the liquid phase and diminish rapidly as the gus is heated and its ‘temperature rises farther and farther above the saturation temperature. A gas approaches the ‘deal state when it reaches a condition such that the interaction between the molecules and hence, internal friction, is negligible. ‘Although no such thing as an ideal or perfect 8 actually exists, many gases, such as air, nitrogen, hydrogen, helium, etc., so closely approach the ideal condition that any errors which may result from considering them to be ideal are of no consequence for all practical Purposes. ‘Although it is important that the student of refrigeration understand and be able to apply the laws of perfect gases, it should be under- stood that gases as they normally occur in the mechanical refrigeration cycle are close to the saturation curve, that is, they are vapors, and do not even approximately approach the condition of an ideal or perfect gas.* They follow the gas laws in only a very general way, * A vapor is sometimes defined as a gas at condition close enough to the saturation curve 30 that it docs not follow the ideal gas laws even approximately. 32 PRINCIPLES OF REFRIGERATION. and therefore the use of the gas laws to deter- ‘mine the pressure-volume-temperature relation- ships of such vapors will result in considerable inaccuracy. In working with vapors, itis usually necessary to use values which have been deter- mined experimentally and are tabulated in saturated and superheated vapor tables. These tables are included as a part of this textbook and are discussed later. 43-16. Processes for Ideal Gases. A gas is said to undergo a process when it passes from some initial state or condition to some final state or condition. A change in the condition of ‘a gas may occur in an infinite number of ways, only five of which are of interest. These are the (1) constant pressure (isobaric), (2) constant volume (isometric), (3) constant temperature (isothermal), (4) adiabatic, and (5) polytropic Processes. In describing an ideal gas, it has been said that the molecules of such a gas are so far apart that they have no attraction for one another, and that none of the energy absorbed by an ideal gas has any effect on the internal potential energy. It is evident, then, that heat absorbed by an ideal gas will either increase the internal kinetic energy (temperature) of the gas or it will Jeave the gas as external work, or both. ‘Since the change in the internal potential energy, AP, will always be zero, the general energy equation for an ideal gas may be written: AQ =AK+aW @-12) In order to better understand the energy changes which occur during the various pro- ‘cesses, it should be kept in mind that a change in the temperature of the gas indicates a change in the internal kinetic energy of the gas, whereas a ‘change in the volume of the gas indicates work done either by or on the gas. 317, Constant Volume Process. When a {gas is heated while it is so confined that ‘volume cannot change, its pressure and temper ture will vary according to Charles’ law (Fig. 33). Since the volume of the gas does not ‘change, no external work is done and AW is equal to zero. Therefore, for a constant volume process, indicated by the subscript», AQ, = AK, G-13) Equation 3-13 is a statement that during a constant volume process all of the energy transferred to the gas increases the internal kinetic energy of the gas. None of the energy leaves the gas as work. ‘When a gas is cooled (heat removed) while its volume remains constant, all the energy removed is effective in reducing the internal kinetic energy of the gas. It should be noted that in Equation 312, AQ represents heat transferred to the gas, SK represents an in- crease in the internal kinetic energy, and AW represents work done by the gas. Therefore, if heat is given up by the gas, AQ is negative, ikewise, ifthe internal kinetic energy of the gas decreases, AK is negative and, if work is done on the gas, rather than by it, AIV isnegative. Hence, in Equation 3-13, when the gas is cooled, boih ‘49 and AK are negative. 318. Constant Pressure Process. If the temperature of a gas is increased by the adit of heat while the gas is allowed to expand so that its pressure is kept constant, the volume of the gas will increase in accordance with Charles’ Jaw (Fig. 3-1). Since the volume of the gus increases during the process, work is done by the gus at the same time that its internal energy is increased. Hence, while one fraction of the transferred energy increases the store of internal kkinetie energy, another fraction of the trans- ferred energy leaves the gas as work. For a constant pressure process, identified by the subscript p, the energy equation may be written 40,=4K, +4", G14) 319, Specific Heat of Gases. The quantity of heat required to raise the temperature of 1 Ib of a gas 1° F while the volume of the gas remains constant is known as the specific heat at a constant volume (C,). Similarly, the quantity of heat required to raise the tempera- ture of 1 Ib of a gas 1° F while the gas expands at a constant pressure is called the specific heat at a constant pressure (C,). For any particular gas, the specific heat at a constant pressure is always greater than the specific heat at a constant volume. The reason for this is easily explained. ‘The quantity of energy required to increase the internal kinetic energy of a gas to the extent that the temperature of the gas is increased 1° F is exactly the same for all processes. Since, during a constant volume process, no work done, the only energy required is that which increases the internal kinetic energy. However, during a constant pressure process, the gas expands a fixed amount for each degree of temperature rise and a certain amount of ‘external work is done. Therefore, during constant process, energy to do the ‘work that is done must be supplied in addition to that which increases the internal kinetic ‘energy. For example, the specific heat of air at a constant volume is 0.169 Btu per pound, whereas the specific heat of air at a constant pressure is 0.2375 Btu per pound. For either process, the increase in the internal energy of the air per degree of temperature rise is 0.169 Btu per pound. For the constant pressure pro- cess, the additional 0.0685 Btu per pound (0.2375 — 0.168) is the energy required to do the work resulting from the volume increase accompanying the temperature rise. ‘The specific heat of a gas may take any value either positive or negative, depending upon the amount of work that the gas does as it expands. 320, The Change in Internal Kinetic Energy. During any process in which the temperature of the gas changes, there will be a change in the internal kinetic energy of the gas. Regardless of the process, when the tempera- ture of a given weight of gas is increased or decreased, the change in the internal kinetic energy can be determined by the equation AK = MCA, — 4) Gi the increase in the internal kinetic energy in Btu _M = the weight in pounds , = constant volume specific heat ‘4 = the final temperature 1, = the initial temperature ‘Nore, The temperature may be in either Fahrenheit or Rankine, since the difference in ‘temperature will be the same in either case as long as the units are consistent, Example 311. The re of 5 Ib Of air is increased by the addition of heat from ‘an initial temperature of 75°F to a final tem- perature of 140° F. If C, for air is 0.169 Buu, ‘What is the increase in the internal energy? ‘Solution. Applying 5 x 0.169 where AK = Equation 3-15, the in- x (140 ~ 75) ‘crease in internal kinetic = 5 x 0.169 x 65 energy AK = 549 Bou THERMODYNAMIC PROCESSES 33 Example 312. Twelve of air are cooled from an initial temperature of 95° F to a final temperature of 72°F. Compute the increase in the internal kinetic energy. Solution, Apply- = 12 x 0.169 ing Equation 3-15, x (72 — 95) the increase in in- = 12 x 0.169 x (—23) ternal kineticenergy ak In Example 3-12, AK is negative, indicating that the gas is cooled and that the internal Kinetic energy is decreased rather than increased. 321. Heat Transferred during a Constant Volume Process. For a constant volume process, since AQ, = 4K, then 46.64 Btu AQ, = MCulty ~ 4) Example 3:13. If, in Example is heated while its volume is What is the quantity of heat G19) 88 during the process? Solution, Apply SQ, = 5 x 0.169 ing Equation 3-16, x (140 ~ 75) the heat transferred 5 x 0.169 x 65 to the gas, = 549 Buu Alternate Solution. From Example ne AK, = 54.9 Btu Since AQ, = AK, 40, = 54.9 Btu Example 3-14, If, in Example 3-12, the air is cooled while its Volume remains constant, ‘what is the quantity of heat transferred to the air during the process? Solution. From Example 3-12, AK, = —46.67 Ba Since AQ, = AK, AQ, = —46.67 Btu In Example 3-14, notice that since AK, is negative, indicating a decrease in the internal kinetic energy, AQ, must of necessity also be negative, indicating that heat is transferred from the gas rather than to it. 3-22, External Work during a Constant Pressure Process. It will now be shown that the work done during a constant pressure ‘process may be evaluated by the equation: W =P, — Vy) Gn 34 PRINCIPLES OF REFRIGERATION ‘where W = the work done in foot-pounds P = the pressure in psfa ¥%, = the final volume in cubic feet = the inital volume in cubic feet ‘Assume that the piston in Fig. 3-1c has an area of A square feet and that the pressure of the gas in the cylinder is P pounds per square foot. Then, the total force exerted on the top of the piston will be PA pounds, or FaPxa ‘Assume now that the gas in the cylinder, having an initial volume Vz, is heated and allowed to ‘expand to volume V, while its pressure is kept constant. In doing so, the force PA acts through the distance 1 and work is done. Hence, WePxAxt but since Ax1=(%-W ‘then W =P, - V3) Example 315. One pound of air having an initial volume of 13.34 cu ft and an initial temperature of 70° F is heated and allowed to ‘expand at a constant pressure of 2117 psfa to a final volume of 15 cu ft. Determine the amount of external work in foot-pounds. Solution. A = 2117 x Equation 30P tte work in” GS 13.39 foot-pounds W = 2117 x 1.66 = 3514 feb In Equation 3-12, AW is always given in heat ‘energy units. By application of the mechanical energy equivalent (Section 3-16), W in foot- pounds may be expressed as AW in Btu. The Telationship is G-18) G19) Express the work done in terms of heat energy units. ‘Solution. Applying Equation asia 38, the work fr Bte HF Te = 452 Bu 323. Heat Transferred during a Constant Pressure Process. According to Equation 3-14, AQy, the total heat transferred to a gas during a constant pressure process is equal to the sum of AK,, the increase in internal kinetic energy, and AW,, the heat energy equivalent of the work of expansion. Example 317. Compute the total heat ‘energy transferred to the air during the constant [Pressure process described in Example 3-15. Solution. Con verting 70°F to degrees Rankine, Applying coat, Equation 3-5, to determine the final temperature, Applying Equa- AK = 1 x 0.169 tion 3-15, the in- x (596 ~ 530) rease in’ internal = 1x 0.169 x 66 Kinetic energy, 1.154 Btu From Example 3-15 and 3-16, AW, = 4.52 Bu Applying Equa- AQ, = 11.15 + 4.52 tion 3-14, 5.67 Bou. Since the specific heat at a constant pressure , takes into account not only the increase in internal energy per pound but also the work done per pound per degree of temperature rise during @ constant pressure expansion, for the constant pressure process only, AQ, may be determined by the following equation: AQ, = MC, — 4) 20) Hence, an alternate solution to Example 3-17 is, Applying Equation AQ, = 1 x 0.237 320, x ($96 — 530) = 1 x 02375 x 66 = 15.67 Bu 324, Pressure-Volume (PV) Diagram. Equation 3-8 is a statement that the thermo- dynamic state of a gas is adequately described bby any two properties of the gas. Hence, using any two properties of the gas as mathematical coordinates, the thermodynamic state of a gas at any given instant may be shown as a point on a chart. Furthermore, when the conditions ‘under which a gas passes from some initial state to some final state are known, the path that the ‘process follows may be made to appear as a line fon the chart. ‘The graphical representation of a process or cycle is called a process diagram or a cycle diagram, respectively, and is a very useful tool in the analysis and solution of cyclic problems. Since work is a function of pressure and volume, when it is the work of a process or cycle which is of interest, the properties used as, coordinates are usually’ the pressure and the volume. When the pressure and volume are used as coordinates to diagram a process or cycle, it is called a pressure-volume (PV) diagram. To illustrate the use of the PV diagram, a pressure-volume of the process de- scribed in Example 3-15 is shown in Fig. 3-4. Notice that the pressure in psfa is used as the vertical coordinate, whereas the volume in cubic feet is used as the horizontal coordinate, In Example 3-15, the initial condition of the gasis such that the pressure is 2117 psfa and the Volume is 13.34 cu fl. To establish the initial state of the gas on the PV chart, start at the origin and proceed upward along the vertical pressure axis to the given pressure, 2117 psfa. Draw a dotted line parallel to the base line through this point and across the chart. Next, from the point of origin proceed to the right along the horizontal volume axis to the given volume, 13.34 cu ft. Through this point draw a vertical dotted line across the chart. The inter- section of the dotted lines at point 1 establishes the initial thermodynamic state of the gas. ‘According to Example 3-15, the gas is heated and allowed to expand at a constant pressure until its volume is 15 cu ft. Since the pressure remains the same during the process, the state point representing the final state of the gas must fall somewhere along the line of constant pressure already established, ‘The exact point ‘on the pressure line which represents the final state 2 is determined by the intersection of the line drawn through the point on the volume axis that identifies the final volume. In passing from the initial state 1 to the final state 2 the air passes through a number of intermediate thermodynamic states, all of which can be represented by points which will fall along line 1 to 2. Line I to 2, then, represents, the path that the process will follow as the ther- modynamic state of the gas changes from 1 to 2, and is the PV diagram of the process described. ‘The area of a rectangle is the product of its THERMODYNAMIC PROCESSES 35 | | 1 r | wl i y Volume (cubic feet) Fig. 34, Pressurevolume diagram of constant Pressure process. Crosshatched area between Process diagram and base line represents external work done during the process. two dimensions. In Fig. 3-4, the area of the rectangle, 1-2-Ve-V; (Crosshatched), is the product ofits altitude P and its base (V, — V4). But according to Equation 3-17, the product P(V, — ¥,) is the external work done during a constant pressure process. It is evident then that the area between the process diagram and the volume axis is a measure of the external work done during the process in foot-pounds. This area is frequently referred to as “the area under the curve. Figure 3-5 is a PV diagram of a constant volume process. Assume that the initial condition of the gas at the start of the process is such that the pressure is 2000 psfa and the volume is 4 cu ft. The gas is heated while its volume is kept constant until the pressure increases to 4000 psfa. The process takes place along the constant volume line from the initial ‘condition 1 to the final condition 2. thas been stated that no work is done during 1 process unless the volume of the gas changes. Examination of the PV diagram in Fig. 3-5 will show that no work is indicated for the constant volume process. Since a line has only the dimension of length, there is no area between the process diagram and the base or volume axis. Hence, no work is done. 345. Constant Temperature Process. ‘According to Boyle's law, when a gas is com- pressed or expanded at a constant temperature, the pressure will vary inversely with the volume. ‘That is, the pressure increases as the gas is 36 PRINCIPLES OF REFRIGERATION. 4500 4000 3500 200 3 B00 : Initial contion po “a == 1500] \ 1000 ! il ° 3s 4s 6 Volume (ube fot) Fig. 35. Pressurevolume diagram of constant volume process. Since there is no area between the process diagram and the volume axis, there Is no work done during a constant volume process, ‘compressed and decreasesas the gas is expanded. Since the gas will do work as it expands, if the temperature is to remain constant, energy with which to do the work must be absorbed from an external source (Fig. 3-28). However, since the temperature of the gas remains constant, all of the energy absorbed by the gas during the process leaves the gas as work; none is stored in the gas as an increase in the internal energy. ‘When a gas is compressed, work is done on the gas, and if the gas is not cooled during the compression, the internal energy of the gas will be increased by an amount equal to the work of compression. Therefore, if the temperature of the gas is to remain constant during the com- pression, the gas must reject to some external body an amount of heat equal to the amount of work done upon it during the compression (Fig. 3-20), There is no change in the internal kinetic ‘energy during a constant temperature process. ‘Therefore, in Equation 3-12, AK is equal to zero and the general energy equation for a constant. temperature process may be written AQ. = Am, G2 3.26. Work of an Isothermal Process. A PV diagram of an isothermal expansion is shown in Fig. 36. In a constant temperature process the pressure and volume both change in Accordance with Boyle's law. The path followed by an isothermal expansion is indicated by line 1 to 2 and the work of the process in foot- pounds is represented by the area 1-2-V-V;. ‘The area, 1-2-%,-V,, and therefore the work of the process, may be calculated by the equa- tion % Warr xine G2) where In = natural logarithm (log to the base ¢) Example 3-18, A certain weight of gas having an inital pressure of 2500 psfa and an "volume of 2 cu ft is expanded isothermally toa volume of 4cu ft. Determine: (@) the final pressure of the gas in psfa, @ the work done in heat energy units. Solution (© By applying Boyle's _ Pi law, Equation 3-6, Va the final pressure Py __ 2500 x 2 “a = 1250 psfa © By applying Equa- tion SOL theta: = 2500 x2 xin ternal work of the = 2500 x 2 x In? process in foot- 2500 x 2 x 0.693 pounds W” = 3465 ftlb By Applying Equa _ M65 tionsl@theworkin~ 778 heatenergyunit ai” = 4.45 Btu Pig. 34, Pressure-volume diagram of constant ‘temperature process. Crosshatched area represents ‘the work of the process. Examp! A certain weight of gas having an initial pressure of 1250 psfa and an initial volume of 4 cu ftis isotherm- ally to a volume of 2 cu ft. Determine: (@ the final pressure in psfa (the work done by the gas in Btu. Solution (@ By applying “hh ‘Boyes law, 1 Equation 3-6, 1280 x4 the final pressure z - = 2500 psta © By Applying Equa- tion 3-22, the ex- = 2500 x 2 x In ternal workof the = 2500 x 2 x In0.5 process in foot- = 2500 x 2 x —0.693 pounds #” = 3465 fab ‘By applying Equation 18, Mos the work in heat 778 energy units SW = —4.45 Blu ‘Notice that the process in Example 3-19 is the exact reverse of that of Example 3-18. Where the process in Example 3-18 is an expansion, the process in Example 3-19 is a compression. Both processes occur between the same two conditions, except that the initial and final conditions are reversed. Notice also that whereas work is done by the gas during the expansion process, work is done on the gas during the compression process. But since the change of condition takes place between the same limits in both cases, the amount of work done in each case is the same. 3:27, Heat Transferred during a Constant Temperature Process. Since there is no change in the temperature during an isothermal process, there is no change in the internal kinetic energy and AK equals zero. According to Equation 3-21, the heat energy transferred ‘during a constant temperature process is exactly equal to the work done in Btu. During an isothermal expansion heat is transferred to the gas to supply the energy to do the work that is done by the gas, whereas during an isothermal ‘compression heat is transferred from the gas so that the internal energy of the gas is not in- creased by the performance of work on the gas. Example 3-20. Determine the quantity of heat transferred to the gas during the constant temperature expansion decribed in Example THERMODYNAMIC PROCESSES 37 ‘Solution. From Example 318, AW = 445 Btu Since, in the isothermal process, AW, equals AQ, AQ, = 4.45 Btu Example 3-21. What is the quantity of heat transferred t6 the gas during the constant ‘temperature process described in Example 3-19. Solution. From Example 3-19, AW = 445 Btu Since AW, equals AQ, AQ, = —4.45 Bu ‘Again, notice that a negative amount of heat is transferred to the gas, indicating that heat in this amount is actually given up by the gas during the process. 3.28. Adiabatic Process. An adiabatic pro- ‘cess is described as one wherein the gas changes its condition without absorbing or rejecting heat, as such, from or to an external body during’the process. Furthermore, the pressure, volume, and temperature of the gas all vary during an adiabatic process, none of them. remaining constant. ‘When a gas expands adiabatically, as in any other expansion, the gas does external work and energy is required to do the work. In the Processes previously described, the gas absorbed the energy to do the work from an external source. Since, during an adiabatic process, no heat is absorbed from an external source, the gas ‘must do the external work at the expense of its ‘own energy. An adiabatic expansion is always accompanied by a decrease in the temperature of the gas as the gas gives up its own internal energy to do the work (Fig. 3-7). When a gas is compressed adiabatically, work is done on the gas by an external body. ‘The energy of the gas is increased in an amount equal to the amount of work done, and since no heat energy is given up by the gas to an external body during the compression, the heat energy ‘equivalent of the work done on the gas is set up as an increase in the internal energy, and the temperature of the gas increases. Because no heat, as such, is transferred to or from the gas during an adiabatic process, AQ, is always zero and the energy equation for an adiabatic process is written as follows: AK, + AW, =0 6-23) ‘Therefore, AW,=-OK, and AK, = —AW, 38 PRINCIPLES OF REFRIGERATION Fig. 37. Pressurevolume diagram of adiabatic Process. An isothermal curve is drawn in for comparison. 3.29. Work of an Adiabatic Process. The ‘work of an adiabatic process may be evaluated 3-24) where k = the ratio of the specific heats of the gas in question, C,/C, Example 3-22, A gas having an initial pressure of 2500 psfa and an initial volume of 2 cu ft is expanded adiabatically to a volume of 4 cu ft. If the final pressure is 945 psfa, determine the external work done in heat energy units. Solution Cy for C, for air The ratio of the specific heats, k 0.2375 Btu/lb = 0.169 Btu/lb 1 ApplyingEquation3.24, (2500 x 2) the work of adiabatic ex- pansion in foot-pounds We ApplyingEquation3-18, __ 3005 the work in heat energy“ 77§ ‘units AW, = 3.86 Bu Example 323. A gas having an initial pressure of 945 psfaand an initial volume of 4.cu ft is compressed adiabatically to a volume of 2 cu ft. If the final pressure of the air is 2500 psfa, how much work is done in heat ‘energy units? Solution. From Example 3-22, & for ait 1.406 ‘Applying Equation 3-24, (945 x 4) — the work done in foot- (2500 x 2) pounds W, 1406 = 3780 — 5000 0.406 1220 0.406 =3005 fb Applying Equation 3-18, _ —3005 the workin heat energy ~ 77g” units AW, = -3.86 Bu 3.30. Comparison of the Isothermal and Adiabatic Processes. A comparison of the isothermal and adiabatic processes is of interest. ‘Whenever a gas expands, work is done by the gs, and energy from some source is required to do the work. In an isothermal expansion, all of the energy to do the work is supplied to the {gas as heat from an external source. Since the energy is supplied to the gas from an external source at exactly the same rate that the gas is doing work, the internal energy of the gas neither increases nor decreases and the tempera- ture of the gas remains constant during the process. On the other hand, in an adiabatic expansion there is no transfer of heat to the gas during the process and all of the work of expan- sion must be done at the expense of the internal energy of the gas, Therefore, the internal ‘energy of the gas is always diminished by an amount equal to the amount of work done and the temperature ofthe gas decreasesaccordingly. Consider now isotherms and adiabatic compression processes. In any compression process, work is done on the gas by the com- pressing member, usually a piston, and an amount of energy equal to the amount of work done on the gas is transferred to the gas as work. During an isothermal compression process, ‘energy is transferred as heat from the gas to an ‘external sink at exactly the same rate that work. is being done on the gas. Therefore, the internal ‘energy of the gas neither increases nor decreases uring the process and the temperature of the ‘gas remains constant. On the other hand, during an adiabatic compression, there is no transfer of energy as heat from the gas to an external sink. Therefore, an amount of energy ‘equal to the amount of work done on the gas is set up in the gas as an increase in the internal ‘energy, and the temperature of the gas increases accordingly. 1B. The Polytropic Process. Perhaps the simplest way of defining a polytropic process is by comparison with the isothermal and adia- batic processes. The isothermal expansion, in which the energy to do the work of expansion is, supplied entirely from an external source, and the adiabatic expansion, in which the energy to do the work of expansion is supplied entirely by the gas itself, may be thought of as the extreme limits between which all expansion processes will fall. Then, any expansion process in which the energy to do the work of expansion is supplied ppartly from an external source and partly from the gas itself will follow a path which will fall somewhere between those of the isothermal and adiabatic processes (Fig. 3-8). Such a process is. known as a polytropic process. If during a polytropic’ expansion most of the energy to do the work comes from an external source, the polytropic process will more nearly approach the isothermal. On the other hand, when the greater part of the energy to do the external ‘work comes from the gas itself, the process ‘more nearly approaches the adiabatic. ‘This is true also for the compression process. When a gas loses heat during a compression process, butnotataratesuffcient to maintain the temperature constant, the compression is poly- tropic. ‘The greater the loss of heat, the closer the polytropic process approaches the iso ‘thermal The smaller the loss of heat, the closer the polytropic process approaches the adiabatic. Of course, with no heat loss, the process becomes adiabatic. ‘The actual compression of a gas in a compres- sor will usually very nearly approach adiabatic, compression. This is because the time of com- pression is normally very short and there is not sufficient time for any significant amount of heat to be transferred from the gas through the cylinder walls to the surroundings. Water jJacketing of the cylinder will usually increase the Fig. 3. Pressure-volume diagram of a polytrople process. Adiabatic and isothermal curves are drawn In for comparison. rate of heat rejection and move the path of the ‘compression closer to the isothermal. 332 PVT Relationship during Adiabatic Processes. Since the temperature, pressure, and volume all change during an adiabatic process, they will not vary in accordance with Charles’ and Boyle's laws. The relationship between the pressure, temperature, and volume during an adiabatic process may be evaluated by the following equations: en m=nx a 625) 1 Th=Tx @) 629 A Fy 62 on marx gy G28) Aone gy G2) nenx @y . 6-20) ‘324, Air is ex] adiabati- cally from a volume of 2 cu ft to a volume of 40 PRINCIPLES OF REFRIGERATION 4cu ft. If the initial pressure of the air is 24,000 psfa, what is the final pressure in psfa? ‘Solution. From Table 3-1, k for air Applying Equation = re 200 x (3 3.27; the final pressure A = 24,000 x 0st." = 24,000 x 0.378 = 9072 psfa Example 325. Air is ex adia- batically from a volume of 2 cu ft to a volume of 4 cu ft. If the initial temperature of the air is 600°R, what is the final temperature in degrees Rankine? Solution. From Table 3-1, for air = 1.406 Applying Equation 699 x URED 3.25, the final tempera- ture 7 = 600 x 0.755 =453°R Example 3-26, Air is expanded adia- bbatically from an initial pressure of 24,000 psfa to a final pressure of fa. If the initial temperature is 600° R, what is the final tem- perature? ‘Solution. From Table 3-1, k for a cs ro eal, (soni the final tempera- a ed =F, essa 3.33. Exponent of Polytropic Expansion and Compression. The presssure-tempera- ture-volume relationships for the polytropic Process can be evaluated by Equations 3-25 through 3-30, except that the polytropic ex- pansion or compression exponent 7 is substi- tuted for k. Too, the work of a polytropic process can be determined by Equation 3-24 if nis substituted for k. ‘The exponent » will always have a value somewhere between 1 and & for the particular gas’ undergoing the process.* Usually, the value of m must be determined by actual tet of the machine in which the expansion or com- pression occurs. In some instances average values of m for some of the common gases undergoing changes under more or les standard conditions are given in tables. If the values of two properties are known for both intial and final conditions, the value of n may be calcu lated. ‘The following sample equation shows the relationship: n= EPP) og (17) Example 3-27." Air, having an [Pressure of 24,000 psfa and an initial tempera- ture of 600° R, is expanded polytropically from a volume of 2'ca ft to 8 volume of 4 ed fit the exponent of polytropic expansion is 1.2, G31) determine: (@) The weight of the air in pounds @ The in psfa (© The final ture in degrees Rankine (@) The work done by the gas in Btu (©) The increase in internal energy Cf) The heat transferred to the gas. Solution @ From Table 3-1, R for air = 333 ‘Rearranging and mala applying Equation 3-10, the weight of 74,000 x 2 air M BS x OO =13 * The value of m depends upon the specific heat of the gas during the process. Since the specific beat may take any value, it follows that theoretically ‘may haveany value. In actual machines, however, 1 will neatly always have some value between 1 and k. Broadly. defined, a polytropic process is any ‘process during which the specific heat remains constant. By this definition, all five processes iscussed in this chapter are polytropic processes. It is general practice today to restrict the term polytropicto mean only those processes which follow ‘8 path falling between those of the isothermal and adiabatic processes. ‘The exponents of isothermal ‘and adiabatic expansions or compressions are 1 and k, respectively. Hence, the value of n for the polytropic process must fall between 1 and k, The closer the polytropic process approaches. the Adiabatic, the closer m will approach f, © Applying Equation ‘aye ‘2h the foal = 24,000 x (7 pressure Py = 24,000 x (0.594 = 24,000 x 0.435 = 10,440 psfa © Applying Equation —_ gq, 4? eben 600 x (mH ture T = 600 x 2F* Gps yg lad a2 (@) Applying Equation 3.24, the wor done W Applyit tion 3.1 werk in Bu AW (© From Table 3-1, = 0.169 Btujlb y for air Aj yuation = 1.5 x 0.169 x 2.13 ie lactase (S22) in internal energy = 15 «0169 x AK (-78) = -19.77 Bi (/Applying Equation = = AK + AW 3.9, the heat ‘rans- = =19.77 + 40.10 feral wo the gus 4Q = 20.83 Bur ‘Notice in Example 3-27 that the work done by the air in the polytropic expansion is equiva- ent to 40.10 Btu, Of this amount, 20.33 Btu is supplied from an external source, whereas the other portion, 19.77 Btu, is supplied by the gas itself, thereby reducing the intemal kinetic energy by this amount. PROBLEMS 1. Three pounds of air occupy a volume of eu ft. Determine: (@) The density of the air. Ans. 0.125 Ibfou fe (©) The specific volume. “Ans. 8 cu ft/lb THERMODYNAMIC PROCESSES 41 2. The volume of a certain weight of air is kept Constant while the ture of the air is increased from 55°F to 100° F. If the initial is 25 psig, what is the final of Thenirin pig ‘an 2.47 Pg 3. A certain weight of sir confined in con tainer is cooled from 150°F to 70°F. If the initial pressure of the air is 36.3 psig, what is the final pressure of the airin psig? Ans.29.6 psig 4. One pound of air at at : fas a volume of 1334 cu aa temperate of 70°F. If the air is passed across a heat exchanger and is heated to a temperature of 150" F while its pressure is kept constant, what is the final volume of the air?) Ans. 15.35 cu ft 5. A cylinder of oxygen has a volume of $ cu ft. ‘A gage on the cylinder reads 2200 psi. If the ‘temperature of the is 85° F, what is the ‘weight of the oxygen in the cylinder? Ans. 60.6 Ib 6. In Problem 4, determine: (@ The work done by the air during the heating. Ans, 4254.8 ftlb or 5.47 Btu (© The increase in the internal kinetic energy. Ans, 13,52 Bou (© The quantity of heat transferred to the air. ‘Ans. 19 Btu 7. A certain weight of air having an initial volume of 0.1334 cu ft and an initial tempera- ture of 70° F is drawn into the suction side of ‘an air compressor. If the air enters the cylinder at standard at pressure and is com- pressed iso ly to a final pressure of 150 psia, determine: (@ The weight of air in the cylinder at the start of the compression stroke. ‘Ans, 0.01 Ib (©) The final temperature of the air in degrees Rankine, ‘Ans, 530° R (© The volume of the air at the end of the compression stroke. Ans. 0.0131 cu ft (@) The work of compression in Btu, ‘Ans. 0.843 Btu (©) The increase or decrease in internal energy. ‘Ans, None (f) The energy transferred to the gas di De compteaton aa Sess ee 8 Assume that the air in Problem 7 is com- pressed adiabatically rather than isothermally. (aT nal temper @ 1 re of the air in Rankine. ‘Ans. 1038°R (@) The volume of the air at the end of the compression stroke. Ans. 0.0256 cu ft (© The work of ion in Btu, someon rs 0.86 Bra 4. PRINCIPLES OF REFRIGERATION (@) The increase in the internal kinetic energy. (8) The volume of the air at the end of the ‘Ans. 0.86 Btu compression stroke. Ans. 0.0192 cu ft (©) The heat energy transferred to or from (¢) The work of compression is Btu. the gas during the compression. Ans. 0.85 Btu “Ans. None (4) The decrease in the internal kinetic energy. Ans. 0.45 Btu ‘Assuming thatthe air in Problem 7 is com- olftropically rather than isothermally. _€) The heat energy transferred from the gas wv equals 1.2, compute: during the compression, Ans. 0.40 Btu (2) The final temperature of the air in degrees 10. Compare the results of Problems 7, 8 ‘Rankine. ‘Ans. 779.4°R and 9, 4 Saturated and Superheated Vapors 4, Saturation Temperature. When the temperature of a liquid is raised to a point such. that any additional heat added to the liquid will cause @ part of the liquid to vaporize, the liquid is said to be saturated. Such a liquid known as a saturated liquid and the tempera- ture of the liquid at that condition is called the saturation temperature (Sections 2-31 and 2-32). 42. Saturated Vapor. The vapor ensuing from a vaporizing liquid is called a saturated vapor as long as the temperature and pressure ‘of the vapor are the same as those of the satur- ated liquid from which it came. A saturated vapor may be described also as a vapor at a temperature such that any further cooling of the vapor will cause a portion of the vapor to condense and thereby resume the molecular structure of the liquid state. It is important to understand that the saturation temperature of the liquid (the temperature at which the liquid will vaporize if heat is applied) and the satura- tion temperature of the vapor (the temperature fat which the vapor will condense if heat is removed) are the same for any given pressure and that the liquid cannot exist asa liquid at any temperature above its saturation temperature, whereas a vapor cannot exist as a vapor at any temperature below its saturation temperature.* * Under certain conditions itis possible to ““super- cool” water vapor momentarily below its saturation, temperature. However, this is a very unstable condition and cannot be maintained except momen- tarily. For example, in Fig. 4-1, the water in the heated vesse! is saturated and is vaporizing at 212° F as the latent heat of vaporization is supplied by the ‘burner. ‘The water vapor (steam) rising from the water is saturated and remains at the satura- tion temperature (212" F) until it reaches the condenser. As the saturated vapor gives up heat to the coaler water in the condenser, it condenses back into the liquid state, Since condensation occurs at a constant temperature, the water resulting from the condensing vapor is also at 212° F. The latent heat of vapor- ization, absorbed as the water vaporizes into steam, is given up by the steam as the steam condenses back into water. 43. Superheated Vapor. A vapor at any temperature above its saturation temperature is superheated vapor (Section 2-34). If after vaporization, a vapor is heated so that its temperature is raised above that of the vapor- izing liquid, the vapor is said to be superheated, In order to superheat a vapor it is necessary to separate the vapor from the vaporizing liquid as shown in Fig. 4-2. As long as the vapor remains in contact with the liquid it will be saturated, This is because any heat added to a liquid- ‘vapor mixture will merely vaporize more liquid and no superheating will occur. Before a superheated vapor can be condensed, the vapor must be de-superheated, that is, the vapor must first be cooled to its saturation temperature. Heat removed from a super- heated vapor will cause the temperature of the ‘vapor to decrease until the saturation tempera ture is reached. At this point, any further removal of heat will cause a part ofthe vapor to condense. 44, Subcooled Liquid. If, after condensa- tion, aliquid is cooled so thet its temperature is reduced below the saturation temperature, the liquid is said to be subcooled. Thus, aliquid at any temperature below the saturation tempera- ture and above the fusion point is a subcooled Tiguid. 45. The Effect of Pressure on the Satura- tion Temperature. The saturation tempera- ture of a liquid or a vapor varies with the pressure. Increasing the pressure raises the saturation temperature and decreasing the pres- sure lowers the saturation temperature, For ‘example, the saturation temperature of water at atmospheric pressure (0 psig or 14.7 psia) is 44 PRINCIPLES OF REFRIGERATION 212°, If the pressure over the water is in- creased from 0 psig to 5.3 psig (20 psia), the saturation temperature of the water increases from 212° F to 228° F. On the other hand, if the pressure over the water is reduced from 14.7 pia to 10 psia, the new saturation temperature Of the water will be 193.2°F. Figure 4-3 is a Condenser water out the water at atmospheric pressure is 212° F, the temperature of the water will rise as the water is heated until it reaches 212° F, At this point, ifthe heating is continued, the water will ‘begin to vaporize. Soon the space above the water will be filled with billions and billions of water vapor molecules darting about at high Steam gives up heat to nid water in condenser ‘and condenses ito water Fig. 41, Saturated vapor. ‘graphical representation of the relationship ‘between the pressure and the saturation tempera ture of water. To illustrate the effect of pressure on the saturation temperature of a liquid, assume that water is confined in a closed vessel which is equipped with a throttling valve at the top (Fig. 4-4a). A compound gage is used to deter- mine the exerted in the vessel and two thermometers are installed so that one records the temperature of the water and the other the temperature of the vapor over the water. With the throttling valve wide open, the pressure exerted over the water is atmospheric (0 psig or 14.7 psia). Since the saturation temperature of, velocities. Some of the vapor molecules will fall ‘back into the water to become liquid molecules again, whereas others will escape through the opening to the outside and be carried away by air currents. If the opening at the top of the vessel is of sufficient size to allow the vapor to escape freely, the vapor will leave the vessel at the same rate that the liquid is vaporizing. That is, the number of molecules which are leaving the liquid to become vapor molecules will be exactly equal to the number of vapor molecules which are leaving the space, either by escaping to the outside or by falling back into the liquid. ‘Thus, the number of vapor molecules and the density of the vapor above the will Fig. 42, Superheated vapor. Pyytt Heat added remain constant and the pressure exerted by the vapor will be equal to that of the atmosphere outside ofthe vessel ‘Under this condition the water vapor ensuing. from the vaporizing liquid will be saturated, that i, its temperature and pressure will be the same as that of the water, 212° F and 14.7 psia. ‘The density of the water vapor at that tempera ture and pressure will be 0.0373 lbjeu ft and its specific volume will be 1/0.0373 or 26:8 fb. Regardless of the rate at which the liquid is ‘vaporizing, as long as the vapor is allowed to ‘escape freely to the outside so that the pressure and density of the vapor over the liquid does not change, the liquid will continue to vaporize at 212°F. SATURATED AND SUPERHEATED VAPORS 45 Suppose that the throttling valve is partially closed so that the escape of the vapor from the vessel is impeded somewhat. For a time the equilibrium will be disturbed in that the vapor will not be leaving the vessel at the same rate the liquid is vaporizing. The number of vapor ‘molecules in the space above the liquid will increase, thereby increasing the density and the pressure of the vapor over the liquid and raising the saturation temperature. it is assumed that the pressure of the vapor increases to 5.3 psig (20 psia) before equilibrium is again established, that is, before the rate at which the vapor is escaping to the outside is exactly equal to the rate at which the liquid is vaporizing, the saturation temperature will be 8 7 Fig. 43, Variation in the saturation temperature of ‘water with changesin pressure. PRINCIPLES OF REFRIGERATION @ ve partly closed 20 pla ® Fig. 44. 228° F, the density of the vapor will be 0.0498 Ib/cu ft, and 1 Ib of vapor will occupy a volume of 20.08 cu ft. This condition is illustrated in Fig. 4-46. By comparing the condition of the vapor in Fig. 449 with that of the vapor in Fig. 44a, it will be noted that the density of the vapor is greater at the higher pressure and saturation temperature. Furthermore, itis evident that the pressure and the saturation temperature of liquid or vapor can be controlled by regulating the rate at which the vapor escapes from over the liquid. In Fig. 4-4a, the rate of vaporization will have litle or no effect on the pressure and saturation temperature because the vapor is allowed to escape freely so that the density and pressure of, the vapor over the liquid will neither increase nor decrease as the rate of vaporization is changed. On the other hand, in Fig. 4-46, any increase in the rate of vaporization will cause an increase in the density and pressure of the vapor and result in_an increase in the saturation temperature. ‘The reason is that any increase in the rate of vaporization will necessitate the escape of a greater quantity of vapor in a given length of time. Since the size ofthe vapor outlet is fixed by the throttling action of the valve, the Pressure of the vapor in the vessel will increase Uuntl the pressure difference between the inside and outside of the vessel is sufficient to allow the vapor to escape at a rate equal to that at which the liquid is vaporizing. ‘The increase in pressure, of course, results in an increase in the saturation temperature and in the density of the vapor. Likewise, any decrease in the rate of ‘vaporization will have the opposite effect. The pressure and density of the vapor over the liquid will decrease and the saturation temperature will be lower. Assume now that the throttling valve on the container is again opened completely, as in Fig. 44a, so that the vapor is allowed to escape freely and unimpeded from over the liquid. The density and pressure of the vapor will decrease until the pressure of the vapor is again equal to that of the atmosphere outside of the container, Since the saturation temperature of water at atmospheric pressures 212° F and since a liquid cannot exist as.a liquid at any temperature above its saturation temperature corresponding to its pressure, it is evident that the water must cool If from 228° F to 212° F at the instant that the pressure drops from 20 psia to atmospheric pressure. To accomplish this cooling, a portion of the liquid will “lash” into a vapor. ‘The latent heat necessary to vaporize the portion of the liquid that lashes into the vapor state is supplied by the mass of the liquid and, as a result of supplying the vaporizing heat, the temperature of the mass of the liquid will be reduced to the new saturation temperature, Enough of the liquid will vaporize to provide the required amount of cooling. 44. Vaporization. The vaporization of a liquid may occur in two ways: (1) by evapora- tion and (2) by ebullition or “boiling.” The ‘vaporization of a liquid by evaporation occurs only at the free surface of the liquid and may take place at any temperature below the satura tion temperature. On the other hand, ebullition or boiling takes place both at the free surface and within the body of the liquid and can occur only at the saturation temperature. Up to this point, only ebullition or boiling has been considered. 47. Evaporation. Evaporation is taking place continually and the fact that water ‘evaporates from lakes, rivers, ponds, clothes, ‘tc,, is sufficient evidence that evaporation can and does occur at temperatures below the saturation temperature. Any liquid open to the atmosphere, regardless of its temperature, will gradually evaporate and be diffused into the air. ‘The vaporization of liquids at temperatures below their saturation temperature can be explained in this manner. The molecules of a liquid are in constant and rapid motion, their ‘velocities being determined by the temperature of the liquid. In the course of their movements the molecules are continually colliding with one ‘another and, as a result of these impacts, some ‘of the molecules of the liquid momentarily attain velocities much higher than the average velocity of the other molecules of the mass. ‘Thus, their energy is much greater than the average energy of the mass. If this occurs within the body of the liquid, the high velocity molecules quickly lose their extra energy in t collisions with other “molecules. However, if the molecules attaining the higher than normal velocities are near the surface, they may project themselves from the surface of the liquid and escape into the air to become vapor molecules. (Fig. 45). The molecules so cescaping from the liquid are diffused throughout the air. ‘They occupy the relatively large spaces which exist between the molecules of the air and become a part of the atmospheric air. ‘44. Rate of Vaporization. For any given temperature, some liquids will evaporate faster SATURATED AND SUPERHEATED VAPORS 47 High energy molecules estape from surface ‘of lquid and become ‘vapor molecules Fig. 45. Evaporation from surface of aliquid. than others. Liquids havingthelowest “boiling” points, that i, the lowest saturation temperature for a given pressure, evaporate at the highest rate. However, for any particular liquid, the rate of vaporization varies with a number of factors. In general, the rate of vaporization increases as the temperature of the liquid increases and as the pressure over the liquid decreases. Evaporation increases also with the amount of exposed surface. Furthermore, it will be shown later that the rate of evaporation is dependent on the degree of saturation of the ‘vapor which is always adjacent to and above the liquid. 49. The Cooling Effect of Evaporation. Since it is the higher velocity molecules (those hhaving the most energy) which escape from the surface of an evaporating liquid, it follows that the average energy of the mass is thereby reduced and the temperature of the mass lowered. Whenever any portion of liquid vaporizes, an amount of heat equal to the latent heat of vaporization must be absorbed by that portion, either from the mass of the liquid, from the surrounding air, or from adjacent objects. Thus, the energy and temperature of the mass are reduced as it supplies the latent hheat of vaporization to that portion of the liquid which vaporizes. The temperature of the ‘mass is reduced to 8 point slightly below that of the surrounding media and the temperature difference so established causes heat to flow from the surrounding media into the mass of the liquid. The energy lost by the mass during vaporization is thereby replenished and evapora- tion becomes a continuous process as long as PRINCIPLES OF REFRIGERATION sutra card ety by a 50 at eapoain ‘coins Notes cmat aor press 07m ig At in dion er we @ « o (ay Fig. 46, any of the liquid remains. The vapor resulting from evaporation is diffused into and carried away by the air. 410. Confined Liquid-Vapor Mixtures. When a vapor is confined in a container with a portion ofits own liquid, both the vapor and the liquid will be saturated. To illustrate, assume that an open container is partially filled with water and is stored where the ambient tempera- ture is 10°F (Fig. 4-64). The water will be evaporating at 70°F and, as described in the Previous section, the vapor molecules leaving the liquid will be diffused into the surrounding air so that evaporation will continue until all of the liquid is evaporated, However, if a tightly fitting cover is placed over the container, the vapor molecules will be unable to escape to the outside and they will collect abovetheliquid. Soon the space above the liquid will be so filled with vapor molecules that there will be as many ‘molecules falling back into the liquid as there are leaving the liquid. A condition of equi- librium will be attained, the vapor will be saturated, and no further evaporation will occur. The energy of the liquid will be increased. by the vapor molecules which are returning to the liquid in exactly the same amount that it is diminished by the molecules that are leaving. Since no further cooling will take place by ‘evaporation, the liquid wall assume the tempera- ture of the surrounding air and: heat transfer will cease. (Fig. 4-66). If, at this point, the ambient ture rises to, say, 80° F, heat transfer will again take place between the surrounding air and the liquid. ‘The temperature and average molecular velocity of the liquid will be increased and evaporation will be resumed. The number of mokeules leaving the liquid will again be greater than the number returning and the density and of the vapor above the liquid will be increased. As the density and pressure of the vapor increase, the saturation temperature of the liquid increases. Eventually, when the saturation temperature reaches 80° and is equal to the ambient temperature, no farther heat transfer will occur and evaporation will cease. Equilibrium will have again been established. The density and pressure of the vapor willbe greater than before, the saturation temperature of the liquid-vapor mixture will be higher, and there will be more vapor and less liquid in the container than previously (Fig. 46), Suppose now that the ambient temperature falls to 60° F. When this occurs, heat will flow from the 80°F liq cooler surrounding air. As the liquid-vapor mixture loses heat to the surrounding air, its temperature and average molecular velocity will ‘be decreased and many of the vapor molecules, ng sulficient energy to remain in the vapor state, will fall back into the liquid and resume the molecular arrangement of the liquid state; that is, a part of the vapor will condense. The density and pressure of the vapor will be diminished and the saturation temperature of the mixture will be reduced. When the satura- tion temperature of the mixture falls to 60° F it will be the same as the ambient temperature and no further heat flow will occur. Equilibrium will have been established and the number of molecules re-entering the liquid will exactly ‘equal those which are leaving. At this new ‘condition, the density and pressure of the vapor ‘will be less than before, the saturation tempera- ture will be lower, and since a part of the vapor ‘condensed into liquid, there wall be more liquid and less vapor comprising the mixture than at the previous condition (Fig. 4-64). 411, Sublimation. It is possible for a substance to go directly from the solid state to the vapor state without apparently passing through the liquid state. Any solid substance will sublime at any temperature below its fusion temperature. Sublimation takes place in a manner similar to evaporation, although much slower, in that the higher velocity ‘molecules near the surface escape from the mass into the surrounding air and become vapor molecules. One of the most familiar examples of sublimation is that of solid CO, (dry ice), which, at normal temperatures and pressures, sublimes directly from the solid to the vapor state. Damp wash frozen on the line in the winter time will sublime dry. During freezing weather ice and snow will sublime from streets and sidewalks, etc. SATURATED AND SUPERHEATED VAPORS 49° 412. Condensation. Condensation of a vapor may be accomplished in several ways: (1) by extracting heat from a saturated vapor, (2) bby compressing the vapor while its temperature remains constant, or (3) by some combination of these two methods. 413, Condensing by Extracting Heat from Saturated Vapor. A saturated vapor has been previously described as one at a condition such that any further cooling will cause a part of the vapor to condense, ‘This is because a vapor cannot exist as a vapor at any tempera- ture below its saturation temperature. When the vapor is cooled, the vapor molecules cannot maintain sufficient energy and velocity to over- ‘come the attractive forces of one another and remain as vapor molecules. Some of the ‘molecules, overcome by the attractive forces, will revert to the molecular structure of the liquid state, When condensation occurs while the vapor is confined 60 that the volume remains constant, the density and pressure of the vapor will decrease so that there is a decrease in the saturation temperature. If, as in a vapor condenser (Fig. 4-1), more ‘vapor is entering the vessel as the vapor condenses and drains from the vessel as a liquid, the density, pressure, and saturation temperature of the vapor will remain constant and condensation will continue as long as heat is continuously extracted from the vapor. 414. Condensing by Increasing the Pres- sure at a Constant Temperature. When a vapor is compressed at a constant temperature, its volume diminishes and the density of the ‘vapor increases as the molecules of the vapor are forced into a smaller volume. The satura- tion temperature of the vapor increases as the pressure increases until a point is reached where the saturation temperature of the vapor is equal to the actual temperature of the vapor. When this occurs, the density of the vapor will be at a ‘maximum value for that condition, and any further compression will cause a part of the vapor to assume the more restrained molecular structure of the liquid state. Thereafter, condensation will continue as long as com- pression continues so that the density and pressure of the remaining vapor cannot be further increased. If the temperature of the vapor is to remain constant, heat must be removed from the vapor during the compression 50 PRINCIPLES OF REFRIGERATION. (ection 3-25), If heat is not removed from the vapor, the temperature of the vapor will increase and condensation will not occur. ‘A careful analysis of Sections 4-13 and 4-14 will show that in cither case the vapor is brought to a saturated condition before conden- sation begins and that heat is removed from the vapor in order to bring about condensation. Furthermore, the vapor is saturated in each case only when the saturation temperature and the actual temperature of the vapor are the same. In Section 4-13, heat is removed from the vapor at @ constant pressure until the tempera- ture of the vapor falls to the saturation tempera~ ture corresponding to its pressure, whereupon the continued removal of heat causes a part of the vapor to condense. In Section 4-14, the pressure of the vapor is increased while the temperature of the vapor remains constant until the saturation tempera~ ture of the vapor corresponding to the increased. is equal to the actual temperature of the vapor. In both cases, since the vapor must ‘give up the latent heat of vaporization in order to condense, heat must be removed from the ‘vapor. 415. Critical Temperature. The tempera- ture of a gas may be raised to a point such that it cannot become saturated regardless of the amount of pressure applied. ‘The critical ‘temperature of any gas is the highest tempers ture the gas can have and still be condensi by the application of pressure. The critical temperature is different for every gas. Some _guses have high critical temperatures while the “eriticat temperatures of others are relatively low. For example, the critical temperature of water vapor is 706°F, whereas the critical ture of air is approximately —225° F. 416. Critical Pressure. Critical pressure is the lowest pressure at which a substance can ture. ‘17. [piportant Properties of Gases and Vapors. Although a gas or vapor has many properties, only six are of particular importance in the study of refrigeration. These are pressure, temperature, volume, enthalpy, internal energy, and entropy. Pressure, temperature, and volume are ‘called measurable properties be- ‘cause they can actually be measured. Enthalpy, internal energy, and entropy cannot be measured. ‘They must be calculated and are therefore known as calculated properties. Pressure, temperature, volume, and internal energy have already been discussed to some extent, A discussion of enthalpy and entropy follows. 418. Enthalpy. Enthalpy is a calculated property of matter which is sometimes loosely defined as “total heat content.” More specif- cally, the enthalpy 27 ofa given mass of material at any specified thermodynamic condition is an expression of the total heat which must be transferred to the material to bring the material to the specified condition from some initial condition arbitrarily taken as the zero point of enthalpy. ‘Whereas the total enthalpy represents the enthalpy of M pounds, the specific enthalpy ‘his the enthalpy of 1 Ib. Since it is usually the specific enthalpy rather than the total enthalpy which is of interest, hereafter in this text the term enthalpy shall be used to mean specific enthalpy, h, the enthalpy of 1 1b. ‘Since litte is known about the specific heat or the other properties of materials at low tempera- tures, it is not possible to determine absolute values for the calculated properties. For this reason, values for the calculated properties must be determined from some arbitrarily selected zero point rather than from absolute zero.* For example, the zero point of enthalpy for water and its vapor, steam, is taken as water at 32°F under atmospheric pressure. ‘The enthalpy of 1 Ib of water at 60° F then is the total amount of heat which must be transferred to the water in order to raise the temperature of the water from 32°F to 60°F. According to Equation 2-9, this is 28 Btu (1 x 1 x 28). Hence, based on the assumption that the enthalpy of water is zero at 32° F, the enthalpy of water at 60° F is 28 Btu/lb. Mathematically, enthalpy is defined as A + Ea aut * Since itis required to know the change in the enthalpy” of the working uid. during. proces, father than the absolte enthalpy at some par- ticular condition, the fact that absolute cathalpy cannot be calculated i of litle consequence. oD SATURATED AND SUPERHEATED VAPORS SI Standard atmospheric pressure Fig. 47. Pressure-volume diagram showing the external work done by fluid expansion at 1 1b of water Is vaporized st atmospheric pressure— approximately 59,000 fe Pressure (psta) vs where h = the specific enthalpy in Btu/lb u = the specific internal energy in Btu/lb P = the pressure in psfa = the specific volume in cubic feet J = the mechanical energy equivalent Tt has been demonstrated (Section 3-12) that all the heat transferred to a fluid is not neces- sarily stored in the fluid as an increase in the internal energy of the fluid. In many cases, some part or all or the heat transferred to the fluid passes through the fluid and leaves the fluid as work. In Equation 4-1, that part of the transferred energy which is stored in the fluid as an increase in the internal energy is repre sented by the term x, whereas that part of the transferred heat which leaves the fuid as work is represented by the term Pu/J. Notice that, although the energy represented by the term ‘Po]J, does not increase the internal energy of the fluid and is not stored in the fluid, it neverthe- less represents energy which must be trans- ferred to the fluid in order to bring the fluid to the specified condition from the initial condition at the arbitrarily selected zero point of enthalpy. Furthermore, even though the external work energy is not stored in the fluid, it must pass back through the fluid and be given, up by the fluid as the fluid returns to the initial condition. ‘Consider, for example, the vaporization of 1 1b of water into steam at 212° F under atmos- pheric pressure. The volume of 1 Ib of water at 212° F is 0.01670 cu ft whereas the volume of 1 Ib of steam at 212° F is 26.82 cu ft. Hence, the fluid expands from a volume of 0.0167 cu ft't0 a volume of 26,82 cu ft during the vaporization thereby doing work in expanding against the pressure of the atmosphere. 2105 psfa (14.696 x 144) {nial condition Final condition Specific volume of. Specific volume of Ll of water at 212°F 1b of steam at 212°F 001672 cut Y 268cu fb Ve ‘Volume cubic feet ‘The enthalpy of vaporization (latent heat of ‘vaporization) of water at 212° F is 970.4 Btu. Of this amount, only 897.6 Btu actually increases the internal energy and represents energy in storage in the vapor. The other 72.8 Btu leaves the vapor as the work of expansion and is represented by the term Pr/J. A PV diagram of the vaporization process is shown in Fig. 4-7. 419. Entropy. Entropy, like enthalpy, is a calculated property of matter. The entropy S of a given mass of material at any specified condition is an expression of the total heat transferred to the material per degree of abso- Jute temperature to bring the material to that condition from some initial condition taken as the zero of entropy. Since it is not possible to calculate the absolute value of entropy, entropy values, like those of enthalpy, are based on an arbitrarily selected zero point. The zero points of entropy and enthalpy are the same for any one fluid. Hence, for water and its vapor, steam, the zero point of entropy is taken as water at 32°F. ‘Again, as in the case of enthalpy, it is the specific entropy 5 rather than the total entropy 'S which is useful. Therefore, in this book, the term entropy shall be used’ to mean specific, entropy s rather than the total entropy S. It has been shown (Section 3-22) that the mechanical energy or work of a process can be expressed as the product of the change in volume and the average absolute pressure. Likewise, it is often convenient to express the heat energy transferred during a process as the product of two factors. The concept of entropy makes this possible. The heat energy trans- ferred during a process can be expressed as the product of the change in entropy and the 52. PRINCIPLES OF REFRIGERATION 672*R(2I2*F + 460) ‘Resolute temperature (78) 2 Ss Se Entropy (Btu/b/"R) average absolute temperature,* Mathematically the relationship is expressed by the following equations: AQ = As x Tm 2) as = 22 3) 4g Tm = a) where AQ = the heat energy transferred in Btu ‘Bs = the change in entropy in Btu per pound per *R Tm = the average absolute temperature in°R On a pressure-volume diagram (Fig. 4-7), the “area under the curve,” which is the product of the change in volume and the average abso- Jute pressure, represents the work of the process. Similarly, on a temperature-entropy diagram (Gemperature plotted against entropy), the “area under the curve,” which is the product, of the entropy change and the average absolute temperature, represents the heat transferred during the process (Fig. 4-8). ‘Although the mathematical treatment of entropy is not required in the study of refrigera~ tion and is beyond the scope of this book, itis. important to note that according to Equation 42 the entropy changes only when heat is transferred during the process. If there is no hheat energy transfer, there is no change in the entropy. The heat energy transfer may occur either to or from an external source or sink or it © The average absolute temperature is not merely the mean of the initial and final temperatures of the process, but is the average of all of the absolute ‘emperatures through which the process passes. Fig. 48 ‘Specific entropy of saturated steam at 212°F 17566 Btu/b/*R may take place entirely within the fluid itself as a result of internal friction. However, the entropy of a fluid is not affected by external ‘work done either by or on the fluid. ‘Thus in a frictionless (occurring without either internal or external friction), adiabatic (no heat transfer to oF from an external body) compression, as in the ideal compression of the refrigerant vapor in a reftigeration compressor, the entropy of the fluid will remain the same or constant. 420. Vapor Tables. It has been stated previously that a vapor does not approach the Condition of an ideal gas because of the inter- molecular forces which exist between the molecules of the vapor. Therefore, internal friction is present whenever a vapor undergoes ‘change of condition so that the various proper- ties of a vapor at the different conditions cannot, be determined by applying the laws of ideal gases. ‘The properties of vapors at various conditions have been determined by experiment for all common vapors and these data are published in the form of tables. Separate tables are used for saturated and superheated vapors. 421, Saturated Vapor Tables. Saturated vapor tables (Fig. 49) dea! only with saturated liquids and vapors, and usually give values for the following properties: (1) temperature, (2) pressure, (3) specific volume, (4) enthalpy (specific), and (5) entropy (specific). Normally, the temperature in degrees Fahrenheit is listed in the extreme left-hand column, The pressure is given in the second and third columns, followed by the specific volume in cubic feet for both the liquid and the vapor in the fourth and fifth columns, respectively. Some tables list the density in addition to or in place of the SATURATED AND SUPERHEATED VAPORS 53 Properties of Saturated Steam ‘Absolute Pressure | Specific Volume Enthalpy Entropy Temp.| Sat sat. | sat. sat, | Sat. Sat. oF, | Psi, | in. Hig, | liquid, |Evap.J vapor tiquid,| Evap., vapor] liquid| Evap. | vapor, ol op P | % | o% | by | he | be | S| Se | Se o}] ® | ®& | ® 1h] 6] M1] & | am] aD | ay 200 | 11.526 | 23.467 [0.01663 |33.62 |33.64 | 167:99| 977. | 1145.9] 0.2938) 1.4824 1.7762 202 | 12011 | 24455 | 0.01665 |32.35 | 32.37 | 170.00] 976.6 | 1146.6] 0.2969) 1.4760] 1.7729 204 | 12512 | 25.475 |0.01666 |31.14 |31.15 | 172.02] 975.4 | 1147.4 0.2999) 1.4697] 1.7696 206 | 13.031 | 26531 | 0.01667 | 2997 29199 | 174.03] 9742 | 1148.2 0.3029] 1.4634 1.7663 208 | 13.568 | 27.625 | 0.01669 |28.86 | 28.88 | 176.04] 972.9 | 1148.9] 0.3059] 1.4571 1.7630 210 | 14.123 | 28.755 | 0.01670 | 27.80 |27.82 | 178.08] 971.6 | 1149.7] 0.3050| 1.4508] 1.7598, 212 | 14,696 | 29.922 | 0.01672 |26.78 | 26.20 | 180.07] 970.3 | 1150.4] 0.3120] 1.4446) 1.7566 214 | 15.289 | 31.129 | 0.01673 |25.81 |25.83 | 182.08] 969.0 | 1151.1] 0.3149] 1.4385] 17534 216 | 15.901 | 32375 | 0.01674 |24.88 | 24.90 | 184.10] 967.8 | 1151.9] 0.3179] 1.4323) 1.7502 218 | 16.533 | 33.662 | 0.01676 |23.99 | 24.01 | 186.11] 966.5 | 1152.6] 0.3209] 1.4262] 1.7471 220 | 17.186 | 34.992 | 0.01677 | 23.13 | 23.15 | 188.13] 965.2 | 1153.4) 0.3239) 1.4201] 1.7440 222 | 17861 | 36.365 | 0.01679 | 2231 |22'33 | 190.13] 963.9 | 1154.1] 0.3268] 1.4141] 1.7409 224 | 18.557 | 37.782 | 0.01680 | 21.53 |21.35 | 192.17] 962.6 | 1154.8] 0.3298) 1.4080] 1.7378 226 | 19.275 |_39.244 | 0.01682 | 20:78 | 20.79 | 194.18| 961.3 | 1155.5} 0.3328] 1.4020| 1.7348 a8 | 0.016) {40.753 [0.01683 [20.06 [20.07 [19620] 960.1 [1156.31 0.535717 9ST] TTE 230 | 20.780 | 42.308 | 0.01684 | 19.365] 19.382] 198.23] 958.8 | 1157.0 0.3387] 1.3901] 1.7288 240 | 24.969 | 50.837 | 0.01692 | 16.306] 16.323] 208.34] 952.2 | 1160.5] 0.3531] 3606] 1.7140 250 | 29.828 | 60.725 | 0.01700 | 13.804] 13.821] 218.48] 945.5 | 1164.0 0.3675 13323] 1.6998 260 | 35.429 | 72.134 | 0.01709 | 11.746] 11.763] 228.64| 938.7 | 1167.3] 0.3817| 13043] 1.6860 270 | 41.858 | 85.225 |0.01717 | 10.044] 10.061] 238.84 931.8 | 1170.6 0.3958] 1.2769] 1.6727 Fig. 49. Excerpt from typical saturated vapor able. Reproduced from Thermodynamic Properties of Steam by Keenan and Keyes, published by John Wiley and Sons, 1936, with permission. volume. If the density only is given and the specific volume is wanted, the specific volume is determined by dividing the density into one. Likewise, when the specific volume is given and the density is wanted, the density is found by dividing the specific volume into one (Gection 3-4). ‘Three values for enthalpy A are usually given in the saturated vapor tables: (1) the enthalpy of the liquid (f,), which is the heat required to raise the temperature of the liquid from the temperature at the assumed zero point of enthalpy to the saturation temperature corre- sponding to the pressure of the liquid; (2) the enthalpy of vaporization (t,,), which is the latent heat of vaporization at the pressure and temperature indicated; and (3) the enthalpy of the vapor (f,), which is the sum of the enthalpy ‘of the liquid (,) and the enthalpy of vapor- ization (hy,).. For example, the enthalpy of the liquid (,) for water at 212° F under atmospheric ‘pressure is 180 Btu (I x 1 x 180), whereas the enthalpy of the saturated water vapor at 212° F ‘under atmospheric pressure is 1050 Btu, which is the sum of the enthalpy ofthe liquid (180 Btu) and the enthalpy of vaporization (970 Btu). Two values of entropy are usually given: 4p, the entropy of the liquid ands, the entropy of the vapor, the difference between the two ‘being the change in entropy during vaporizat 54 PRINCIPLES OF REFRIGERATION. Dichlorodifluoromethane (Refrigerant-12) Properties of Superheated Vapor ‘Abs. Pressure 36 Ibjin® | Abs. Pressure 38 Ib/in | “Abs. Pressure 40 Ib/in* | Abs. Pressure 42 ib/in® ‘Temp. | Gage Pressure 21.3 Ibjin* | Gage Pressure 23.3 bfin.*| Gage Pressure 25.3 Ijin | Gage Pressure 27.3 Ibin.* “F | Gat. Temp. 204°F) | Gat. Temp. 232°F) | Gat. Temp. 259° F) | (Sat Temp. 285°F) lv s |v mus |v us |v ows satin] ati 030 @tssen| (os ose (tees | 1.009) 16) oxes10| (0360 (140 (16897) ‘30 | 1140 1017227 | 1.016 8182 0.17126 | 1.019 “81.76 0.17030 | 0967 81.65 0.16939, a | ties 8235 crate | Los 327 ovate | tive 8320 017922 | 0991 ant0 @trast 50 | 1.196 0.17806 0.17706 [TOT HESSOITEIT] 1.016 34.56 0.17521 6 | 1223 86.27 0.18089 O.t7o91 [1095—eTO17E9E-] Todo 86.03 0.17806 70 | 1250 87:74 o.18369 oe272 | 11120 #7.60 o8i78 | 1063 47.51 0.18086 go | 1278 8922 18687 Ouesst | 11144 9.09 oe4ss | 1087 $9.00 O.18365 90 | 1305 90°71 octss2i 0.18826 | 11169 90.58 0.18931 | 1.110 90.50 0.18640 too | 1332 92.22 osi93 0.19096 | 1.194 92.09 0.19004 | 1.134 92.01 0.18913 uo | 1359 93.75 otsaer | 1:285 93.69 o.1s365 93.62 a1s772 | 1.138 93.34 o.lsi86 120 | 1386 95:28 0.19729 | 13310 9522 o.19631 9515 019538 | 118i 95.09 o.19451 130 | 11412 96:82 0.19991 | 1336 96:76 o.989s 9670 0.19803 | 1204 96.64 0.19714 140 | 1439 98.37 0.20286 | 1361 98:32 0.20157 98.26 0.20066 | 1.227 98.20 0.19979, sso | 146s 9993 0.20512 | 1.387 99.89 o20416 99.83 0.20325 | 1.250 99.77 0.20237 160 | 1.492 101.51 0.20770 | 1412 101.47 0.20673, y0l.42 0.20883 | 1274 0.20496 170 | 51 103.11 o21028 | 1437 10307 0.20929 10x02 0.20838 | 1297 102196 0.20751 igo | 1'545 104.72 02127 | 1'462 10467 021183 10463 0.21092 | 1320 104.57 0:21005 190 | 11571 106:34 0.21528 | 1487 106.29 0.21433, 10625 0.21343 | 1343 106.19 0.21256 200 | 1.597 10797 0.21778 | 1.512 10793 0.21681 107.88 0.21892 | 1.365 107.82 0.21508 210 | 1623. 109.61 0.22024 | 1°337 109157 021928 10952 021840 | 1388 021758 220 | 1.650 111.27 022270 | 1'562 11122 022176 Ana? 022085 | rant 0.22000 230 | 1.676 11298 0.22813 | 1387 11289 022419 112'84 029329 | 1434 112180 022244 240 | 1702 114.62 0.22756 | 1.612 114.38 0.22662 11432022872 | 1.487 114.49 022486 250 | 1.728 11631 0.22996 | 1.637 116.28 0.22903 116.21 022813 | 1480 11619 0.22728, 260 | 1.754 148.02 0.25235 | 1.662 11799 023142 11792 023052 | 11502 117.90 022967 290 | 17H0 11976 0.23472 | 1.687 119.71 023379 19.65 024289 | 1524. 119162 0.23204 20 | 1807 12147 0.23708 | 1.712 12145 023616 121.40 023526 | 11587 121.36 023441 290 | 11833. 12322 023042 | 1.737 12320 0.23880 123.15 0.23760 | 1570 123.11 023675 300 0.24083 | 1.673 124.92 0.23994 | 1.592. 124.87 0.23909 Fig. 4-10. Excerpt from typical superheated vapor table. ‘Copyright by E. 1. du Pont de Nemours and Co., Inc. Reprinted by permission, It has been stated previously that the con- dition of a gas or a vapor can be determined ‘when any two ofits properties are known. How- ‘ever, for a saturated liquid or vapor at any one pressure, there is only one temperature that the fluid can have and still satisfy the conditions of saturation. This is true also for the other properties of a saturated liquid or vapor. ‘Therefore, if any one property of a saturated liquid or vapor is known, the value of the other properties can be read directly from the satu- rated vapor table, For instance, assume that the pressure of one pound of dry saturated steam is 20 psia. By locating 20 psia (encircled) in the second column of the abbreviated table in Fig. 4.9 and reading across the table, the values (et off by the heavy lines) for all the other Properties of the vapor at this condition can be obtained. 422. Superheated Vapor Tables. A super- heated vapor table deals with the properties of ‘a superheated vapor rather than those of a saturated vapor, and the arrangement of @ superheated vapor table is somewhat different from that of a saturated vapor table. One ‘common form of the superheated vapor table is illustrated in Fig. 4-10. Before examining the superheated vapor table, it is important to take note of one significant difference between a saturated and a superheated vapor. Whereas, for a saturated vapor at any one pressure there is only one temperature which will satisfy the conditions of saturation, a superheated vapor may have ‘any temperature above the saturation tempera~ ture Corresponding to its pressure. ‘The specific volume, enthalpy, and entropy of a superheated vapor at any one pressure will vary with the temperature. This does not mean that the properties of a superheated vapor are entirely independent of the pressure of the vapor but only that the properties of the superheated ‘vapor at any one pressure will vary with the temperature. As a matter of fact, superheated vapor tables are based on the pressure of the vapor, and before the properties of a super- heated vapor can be determined froma table, the pressure of the vapor or one of the properties of the vapor at saturation must be known. When one of the properties of the vapor at saturation is known, the pressure of the vapor cean be found by consulting a saturated vapor table. In addition to the properties of the super- heated vapor at various temperatures above the saturation temperature corresponding to the pressure, superheated vapor tables usually list ‘some or all of the properties of the vapor at the saturation temperature. For example, in Fi 410, the absolute and gage pressures, along with the saturation temperature corresponding to these pressures, are given at the head of the table. ‘The first readings in the body of the table (italicized) lists the specific volume, the enthalpy, and the entropy of the vapor at saturation. The specific volume, enthalpy, and entropy of the superheated vapor at various ‘temperatures above the saturation temperature make up the body of the table. ‘Notice that the temperature of the superheated vapor, given in the éxtreme left-hand column, is listed in 10°F increments. Example 41. One pound of superheated Refrigerant-12 vapor is at a temperature of 50° F and its is 40 psia. From the abbreviated table in Fig. 4-10, determine: (@ The temperature, volume, enthalpy, and entropy of the vapor at saturation (© The volume, enthalpy, and entropy of the ‘vapor at the superheated condition (©) The degree of superheat of the vapor in degrees Fahrenheit SATURATED AND SUPERHEATED VAPORS (@ The amount of superheat in the vapor in Bu 3 (©) The change in the volume during the superheating (J) The change in entropy during the super- heating Solution (@ From the head of the table, the satura- tion temperature corresponding to ‘opin From the body of the table (Grst reading, _itali- cized),thespecific volume of the vapor at satura- tion ‘The enthalpy of the vapor at satura- tion ‘The entropy of the ‘vapor at satura tion = 1.009 cu ft/lb, = 81.16 Btu/lb = 0.16914 Btu/lb/* R (® From the body ‘of the table, the properties of the vapor superheated to 50° F (offset by heavy lines in Fig. 10) the. specie volume = 170 cu Theenthalpy = 84.65 Bib ‘The entropy = 0.17612 Btu/lb/? R (© The superheated. ‘empeate = s00'F The temperature at sea Laser ‘he dere of super ata te vapor in dees Pre cenheit =40F (@) The enthalpy of ‘he ‘sperbeated var = 2465 Bb Theeathapy ofthe vapor af face ton = 8116 Buf The amount of peat the wera Bia = 52GB 56 PRINCIPLES OF REFRIGERATION © The entropy of the superheated vapor The entropy of the ‘vapor at satura- tion ‘The change in entropy during the supereatng (P)The volume of the = 0.17612 Buu/ib/*R superheated vapor The volume of the vapor at satura- = 0.16914 Bru/ib/*R tion ‘The change in vol- ume during the = TOOT BUTE R ——_superheating 1.070 cu fejlb 1.009 cu fib 0.061 eu fyb 5 Psychrometric Properties of Air Composition of Air. Air is a mecha- nical mixture of gases and water vapor. Dry air (air without water vapor) is composed chiefly of nitrogen (approximately 78% by volume) and oxygen (approximately 21%), the remaining 1 % being made up of carbon dioxide and minute quantities of other gases, such as hhydrogen, ‘helium, neon, argon, etc. With regard to these dry air components, the com position of the air is practically the same everywhere. On the other hand, the amount of water vapor in the air varies greatly with the particular locality and with the weather con- ditions. Since the water vapor in the air results primarily from the evaporation of water from the surface of various bodies of water, atmos- pheric humidity (water vapor content) is ‘greatest in regions located near large bodies of ‘water and is less in the more arid regions. Since all air in the natural state contains a certain amount of water vapor, no such thing as “dry ait” actually exists, Nevertheless, the concept of “dry air” is a very useful one in that it greatly simplifies psychrometric calculations. Hereafter in this book the term “dry ait” is used to denote air without water vapor, whereas the term “air” is used to mean the natural snixture of “dry air” and water vapor. 52. Air Quantities. Air quantitics may be stated either in units of volume (cubic feet) or in ‘units of weight (pounds) so that the need for Exam) converting air quantities from one unit of 95°F is ci ‘measure to the other occurs frequently. 7 ‘The volume occupied by any given weight of air depends upon the pressure and temperature of the air, and varies inversely with the baro- metric pressure and directly with the absolute temperature. Air very nearly approaches the condition of an ideal gas and will follow the gas Jaws with sufficient accuracy for all practical purposes. Therefore, the volume occupied by any given weight of air at any given pressure and temperature can be determined by applying Equation 3-10. Example 5-1. Determine the volume oc- ccupied by 1 Ib of air having a temperature of 70" F at standard sea level pressure (14.7 psia).. Solution. Rearranging MXR XT and applying “Equation P Fi, ne Po 1x 533 x (10 + 460) Tae = 1334 cuft Example 5-2. Determine the volume of the air in Example 5-1 ifthe barometric pressure is 126 psa Solution. ‘Applyis Equation 10 1 x 533 x (70 + 460) 126x144 = 1557 cuft ve Example 5-3, Determine the volume of the air in Example 5-1 if the temperature of the air is 100° F, Solution. Ay 1x 53.3 x Equation 3-10, Frm (100 + 460) "a7 x 14 = 14.1080 The relationship between the volume and the i i condition oy r (5-2) Mar (2) ‘where M = the weight of air in pounds V = the volume of M pounds of air in cibic feet vm the specific volume of the air in cubic feet per pound 5-4, Air at a temperature of irculated over a all ee the ate of 2000 cu fi/min (cfm). 58 PRINCIPLES OF REFRIGERATION volume of the air is 14.38 cu ft/lb, determine the weight of air passing over the coil in pounds per hour. Solution. Applying sunton 3, te weet of air passing over the cooling coll © Multiplying by 60 min M = 139.1 x 60 ee = 8346 Ibjhr 5-3. Standard Air. Because of the difference in the volume of any given weight of air at various temperatures and pressures, an air standard has been established for use in the rating of air handling equipment so that al ‘equipment is rated at equal conditions. Dry ai having a specific volume of 13.34 cu ft per pound for a density of 0.07496 (0.075) Ib per cu ft (1/13.34) is defined as standard air. Air at a temperature of 70° F and at standard sea level pressure has this specific volume and density (Gee Example 5-1). ‘A given volume of air at any condition can be converted to an equivalent volume of standard ‘air by applying the following equation: 2000 M = Ta38 = 139.2 Ib/min a tax® (5-3) where ¥, = the equivalent volume of standard air V4 = the actual volume of the air at any given condition 4 = the specific volume of the air at the given condition , = the specific volume of standard air (13,34 cu ftfb) Example 5-5, For the air in Example 5-4, determine the equivalent volume of standard air. Solution. Applying Equation 5-3, the equiv. _ 2000 x 14.38 alent volume of standard 13.34 air V, = 2155 cfm 54, Dalton’s Law of: Partial Pressure. Dalton’s law of partial pressures states in effect that in any mechanical mixture of gases and ‘vapors (those which donot combine chemically): () each gas or vapor in the mixture exerts an. individual partial pressure which is equal to the pressure that the gas would exert if it occupied the space alone and (2) the total pressure of the gaseous mixture is equal to the sum of the partial pressures exerted by the individual {gases or vapors. Air, being a mechanical mixture of gases and water vapor, obeys Dalton's law. Therefore, the total barometric pressure is always equal to the sum of the partial pressures of the dry gases, and the partial pressure of the water vapor. Since psychrometry is the study of the properties of air as affected by the water vapor content, the individual partial pressures exerted by the dry gases are unimportant and, for all practical purposes, the total barometric pressure may be considered to be the sum of only two pressures: (1) the partial pressure exerted by the dry gases and (2) the partial pressure exerted by the water vapor. 5.5. Dew Point Temperature. It is im portant to recognize that the water vapor in the air is actually steam at low pressure and that, this low pressure steam, like high pressure steam. will be in a saturated condition when its ‘temperature is the saturation temperature corre~ sponding to its pressure. Since all of the com- ponents in a gaseous mixture are at the same temperature, it follows that when air is at any ‘temperature above the saturation temperature ‘corresponding to the partial pressure exerted by the water vapor the water vapor in the air will bbe superheated. On the other hand, when air is at a temperature equal to the saturation temperature corresponding to the partial pressure of the water vapor, the water vapor in the air is saturated and the air is said to be saturated (actually it is only the water vapor which is saturated), The temperature at which the water vapor in the air is saturated is known as the dew point temperature of the air. Ob- viously, then, the dew point temperature of the air is always the saturation temperature corre- sponding to the partial pressure exerted by the water vapor. Hence, when the partial pressure exerted by the water vapor is known, the dew point temperature of the air can be determined from the steam tables. Likewise, when the dew point temperature of the air is known, the partial pressure exerted by the water vapor can. bbe determined from the steam tables. Example 5-6. Assume that a certain quan- tity of air has a temperature of 80° F and that the partial pressure exerted by the water vapor in the air is 0.17811 psia. Determine the dew point temperature of the air. Solution. From Table 4-1, the saturation ture of steam corresponding to a pres- sure of 0.17811 psia is $0° F. Therefore, 50° F is the dew point temperature of the air. Example 5-7. A certain quantity of air has ‘a temperature of 80°F and a dew point tem- perature of 40° F, Determine the partial pres- sure exerted by the water vapor in the air. Solution. From Table 41, the saturation pressure corresponding to 40° F is 0.12170 psia and therefore 0.12170 psia is the partial pressure exerted by the water vapor. Tt has been shown (Section 4-5) that the pressure exerted by any vapor is. directly Proportional to the density (weight per unit volume) of the vapor. Since the dew point ‘temperature of the air depends only on the partial pressure exerted by the water vapor, it follows that, for any given volume of air, the dew point temperature of the air depends only upon the weight of water vapor in the air. As Tong as the weight of water vapor in the air remains unchanged, the dew point temperature of the air will also remain unchanged. If the amount of water vapor in the air is increased or decreased, the dew point temperature of the air will also be increased or decreased, respectively. Increasing the amount of water vapor in the ait will increase the pressure exerted by the water vapor and raise the dew point temperature. Likewise, reducing the amount of water vapor in the air will reduce the pressure of the water vapor and lower the dew point temperature. 546. Maximum Water Vapor Content. ‘The maximum amount of water vapor that can bbe contained in any given volume of air depends ‘only upon the temperature of the air. Since the amount of water vapor in the air determines the pressure exerted by the water vapor, it fent that the air will contain the maximum. amount of water vapor when the water vapor in the air exerts the maximum possible pressure Since the maximum pressure that can be exerted by any vapor is the saturation pressure corresponding to its temperature, the air will contain the maximum weight of water vapor ‘when the pressure exerted by the water vapor {sequal to the saturation pressure corresponding. to the temperature of the air. At this condition the temperature of the air and the dew point temperature will be one and the same and the air PSYCHROMETRIC PROPERTIES OF AIR 59 ‘willbe saturated. It is important to notice that the higher the temperature of the air, the higher is the maximum possible vapor pressure and the ‘greater is the maximum possible water vapor content. 5-1, Absolute Humidity. The water vapor in the air is called humidity. The absolute humidity of the air at any given condition is defined as the actual weight of water vapor contained in 1 cu ft of air at that condition. ‘Since the weight of water vapor contained in the air is relatively small, it is often measured in ‘grains rather than in pounds (7000 grains equal 1b). 58 The Psychrometric Tables. It was shown in Section 5-5 that the actual weight of ‘water vapor contained in a unit volume of air is solely a function of the dew point temperature of the air. Because of this fixed relationship between the dew point temperature and the absolute humidity of the air, when the value of one is known, the value of the other can be readily computed. ‘The absolute humidity of air at various dew point temperatures is listed in Tables 5-1 and 5.2. The dew point temperatures are listed in column (1) of the tables, and the absolute humidity corresponding to each of the dew point temperatures is given in columns (4) and (5). ‘The values given in column (4) are in pounds of water vapor per cubic foot of air, ‘whereas the values given in column (5) are in grains of water vapor per cubic foot of air. Too, the partial pressure (saturation pressure) of the vapor corresponding to each dew point temperature is given in inches of mercury in column (2) and in pounds per square inch in column (3). 5.9, Relative Humidity. Relative humidity (RH), expressed in percent, is the ratio of the actual weight of water vapor per cubic foot of air relative to the weight of water vapor con- tained in a cubic foot of saturated air at the same temperature, viz: Actual weight of ‘water vapor per cubic foot of air Weight of water vapor in I cu ft of saturated air at the same temperature Relative humidity = x 100 (5-4) 60 PRINCIPLES OF REFRIGERATION For instance, if air at a certain temperature contains only half as much water vapor per ‘cubic foot of air as the air could contain at that temperature if it were saturated, the relative humidity of the air is 50%. "The relative humidity of saturated ar, ofcourse, is 100%. Example 5-8. Air at a temperature of 80°F has a dew point temperature of 50°F. Determine the relative humidity. ‘Solution. From Table 5-2, absolute humidity correspond- ing to dew point tem- perature of 50° F Absolute humidity of saturated air at 80°F Applying Equation 4, de eiavebumi ity of the air = 4.106 grainsfeu ft = 1104 grainsfou ft 106 = Foog * 100 =371 Example 5-9, Determine the relative hu- midity of the ait in Example 5-8, if the air cooled to 60° F. (Note: the dew point tempera- ture of the air does not change because the ‘moisture content does not change.) Solution. From Table 5-2, absolute humidity corresponding to dew point of 50°F Absolute humidity of saturated air at 60° F Applying Equation 5-4, the relative humidity of the air = 4.106 grainsjeuft = 5.795 grainsjeuft = 20.8% 5-10, Specific Humidity. The specific humid- ity is the actual weight of water vapor mixed with 1 1b of dry air and is usually stated in ‘grains pet pound, that is, grains of water vapor per pound of dry air. For any given barometric pressure, the specific humidity is a function of the dew point temperature alone. The specific humidity of air at various dew point tempera- tutes is listed in Columns 6 and 7 of Tables 5-1 and 5-2. In Column 6, the specific humidity is given in pounds of water vapor per pound of dry air, whereas in Column 7 the specific humidity is given in grains of water vapor per pound of dry air. Since the speciic humidity correspond- ing to any given dew point temperature varies with the total barometric pressure, the values given in Tables 5-1 and 5-2 apply only to air at standard barometric pressure. ‘The specific humidity of the air at any given dew point temperature increases as the total barometric pressure decreases and decreases a8, the total barometric pressure increases. The reason for this is easily explained, It has been shown (Examples 5-1 and 5-3) that the volume ‘occupied by 1b of air increases as the total barometric pressure decreases. Since the density of a vapor varies inversely with the volume, it follows that the weight of water vapor required to produce a given vapor density and vapor pressure increases as the volume of the air increases. Likewise, as the volume occupied by 1 1b of air diminishes, the weight of water vapor required to produce a certain vapor density and vapor pressure also diminishes. S-Il, Percentage Humidity. Percentage humidity is defined as the ratio of the actual ‘weight of water vapor in the air per pound of dry air to the weight of water vapor required to saturate completely 1 Ib of dry air at the same temperature. Percentage humidity, like relative humidity, is given in percent. Notice, however, that percentage humidity is associated with the ‘weight of water vapor per unit weight of air, whereas relative humidity is associated with the weight of water vapor per unit volume of air. For this reason the percentage humidity varies with the total barometric pressure, whereas relative humidity does not. Example 5-10, Air at stondard sea level pressure has a temperature of 80°F and a dew point temperature of 50°F. Determine the specific humidity and percentage humidity of the air. Solution. From Table 5.2, the specific humidity Of the air in grains per pound corresponding to 50° F dew point tem- perature (Column 7) Specific humidity of saturated air at 80°F (Column 7) Percentage humidity 33.38 grainsylb = 155.50 grains/Ib Nore. Compare this value with the relative humidity obtained in Example 5-8. 5-12, Dry Bulb and Wet Bulb Tempera- tures, The dry bulb (DB) temperature of the Airis the temperature as measured by an ordinary dry bulb thermometer. When measuring the dry bulb temperature of the air, the bulb of the ther- ‘mometer should be shaded to reduce the effects Of direct radiation. ‘The wet bulb (WB) temperature of the air is the temperature as measured by a wet bulb ther- ‘mometer. A wet bulb thermometer is an ordi- nary thermometer whose bulb is enclosed in a wetted cloth sac or wick. To obtain an accurate reading with a wet bulb thermometer, the wick should be wetted with clean water at approxi- mately the dry bulb temperature of the air and the air velocity around the wick should be: tained between 1000 and 2000 ft pec minute. As ‘a practical matter, this velocity can be simulated in stil air by whirling the thermometer about on the end of a chain. An instrument especially esigned for this purpose is the sling psychrom- ‘ter (Fig. 5-1). The sling psychrometer is made up of two thermometers, one dry bulb and one ‘wet bulb, mounted side by side in a protective ‘case which is attached to a handle by a swivel connection so that the case can be easily rotated about the hand, After saturating the wick with clean water, the instrument is whirled rapidly in the air for approximately oneminute, after which time readings can be taken from both the dry bulb and wet bulb thermometers. The process should be repeated several times to assure that the lowest possible wet bulb temperature has been recorded. ‘Unless the ar is 100% saturated, in which case the dry bulb, wet bulb, and dew point tempera- tures of the air will be one and the same, the temperature recorded by a wet bulb thermom- eter will always be lower than the dry bulb temperature of the air. The amount by which the wet bulb temperature is reduced below the dry bulb temperature depends upon the relative humidity of the air and is called the wet bulb depression. ‘Whereas a dry bulb thermometer, being un- affected by humidity, measures only the actual ‘temperature of the ait, a wet bulb thermometer, Ddecause of its wetted Wick, is greatly influenced by the moisture in the air; thus a wet bulb tem- perature isin effect a measure of the relationship ‘between the dry bulb temperature of the air and the moisture content of the air. In general, for PSYCHROMETRIC PROPERTIES OF AIR 61 any given dry bulb temperature, the lower the moisture content of the air, the lower is the wet bulb temperature. The reason for this is easily explained, ‘When unsaturated air is brought into contact with water, water will evaporate into the air at a rate proportional to the difference in pressure between the vapor pressure of the water and the vapor pressure of the air. Hence, when a wet bulb thermometer is whirled rapidly about in ‘unsaturated air, water will evaporate from the ‘wick, thereby cooling the water remaining in the wick (and the thermometer bulb) to some tem- perature below the dry bulb temperature of the air. It is important to recognize the fact that the ‘wet bulb temperature of the air is a measure of the relationship between the dry bulb and dew point temperatures of the air, and as such it provides a convenient means of determining the ‘dew point temperature of the air when the dry 62 PRINCIPLES OF REFRIGERATION bulb temperature is known. Too, it will be shown later that the wet bulb temperature is also fn index of the total heat content of the air. In order to understand why the wet bulb tem= perature isa measure of the relationship between the drybulb and dew point temperatures, aknow- ledge of the theory of the wet bulb thermometer is required. When water evaporates from the wick of a wet bulb thermometer, heat must be supplied to furnish the latent heat of vaporiza- tion. Before the temperature of the water in the wick is reduced below the dry bulb temperature of the air, the source of the heat to vaporize the water is the water itself. Therefore, as water evaporates from the wick, the water remaining in the wick is cooled below the dry bulb tempera- ture of the air. When this occurs, a temperature differential is established and heat begins to flow from the ai to the wick. Under this condition, a part of the vaporization heat is being supplied by the air while the other part is supplied by the water in the wick. As the temperature of the wick continues to decrease, the temperature difference between the air and the wick increases progressively so that more and more of the vaporization heat is supplied by the air and less and less is supplied by the water in the wick ‘When the temperature of the wick is reduced to the point where the temperature difference between the air and the wick is such that the flow of heat from the air is sufficient to supply all of the vaporizing heat, the temperature of the ‘wick will stabilize even though vaporization from the wick continues, ‘The temperature at which the wick stabilizes is called the temperature of adiabatic saturation and is the wet bulb tempera- ture of the air. ‘Through careful analysis of the foregoing, it can be seen that the wet bulb temperature depends upon both the dry bulb temperature and the amount of water vapor in the air. For example, the lower the relative humidity of the air, the greater is the rate of evaporation from ‘the wick and the greater is the amount of heat required for vaporization. Obviously, the greater the need for heat, the greater is the wet bulb depression below the dry bulb temperature. Too, it follows also that the lower the dry bulb tem- ‘perature, the lower the wet bulb temperature for any given wet bulb depression. 5-13. The Heat Content or Enthalpy of ‘Air. Air has both sensible and latent heat, and the total heat content of the air at any condition is the sum of the sensible and latent heat contained therein. The sensible heat of the air isa function of the dry bulb temperature. For any given dry bulb temperature, the sensible heat of the air is taken as the enthalpy of dry air at that temperature as calculated from0” F. Airsensible heat at various temperatures is given in Btu per pound of dry air in Column 10 of Tables 5-1 and-2, Withregard to Column 10, the temperatures listed in Column 1 are used as dry bulb temperatures. Example 5-11, Using Table 5-2, determine the sensible heat in 10 Ib-of air at 80° F. ‘Solution. FromTable ‘5:2, the sensible heat of 1 Ib of air at 80° F For 10 Ib of = 19.19 Btu/lb 10 x 19.19 = 191.9 Bw ‘The quantity of sensible heat added or removed in heating or cooling a given weight of air through a given temperature range may be computed by applying Equation 2-8. The mean specific heat of air at constant pressure is 0.24 Buu/lb. (Although the specific heat of any vapor for gas varies somewhat with the temperature range, the use of a mean specific heat value is sufficiently accurate for all practical purposes.) Example 5-12, Compute the quantity of sensible heat required to raise the temperature of 10 Ib of air from 0° F to 80° F. Solution. Applying Equation 2-8, O, = 10 x 0.24 x (80 — 0) 192 Btu Alternate Solution. From ‘Table 5-2, the sensible heat of 1 Ib of air at 80° F Sensible heat of 1 Ib of = 19.19 Btu/lb air at 0° F = 0 Btu/lb For 1 Ib of air, O, = 19.19 -0 19.19 Btu/tb For 10 Ib of air, Q, = 10 x 19.19 = 191.9 Btu Since all the components of dry air are non- condensable at normal temperatures and pres- ‘sures, for all practical purposes the only latent ‘heat in the air is the latent heat of the water vapor in the air. Therefore, the amount of

You might also like